Download BIOCHEMISTRY Which of the following single

Document related concepts

Basal metabolic rate wikipedia , lookup

Ketosis wikipedia , lookup

Evolution of metal ions in biological systems wikipedia , lookup

Peptide synthesis wikipedia , lookup

Oxidative phosphorylation wikipedia , lookup

Butyric acid wikipedia , lookup

Enzyme wikipedia , lookup

Phosphorylation wikipedia , lookup

Proteolysis wikipedia , lookup

Metalloprotein wikipedia , lookup

Genetic code wikipedia , lookup

Deoxyribozyme wikipedia , lookup

Artificial gene synthesis wikipedia , lookup

Point mutation wikipedia , lookup

Fatty acid metabolism wikipedia , lookup

Fatty acid synthesis wikipedia , lookup

Glycolysis wikipedia , lookup

Hepoxilin wikipedia , lookup

Citric acid cycle wikipedia , lookup

Nucleic acid analogue wikipedia , lookup

Metabolism wikipedia , lookup

Amino acid synthesis wikipedia , lookup

Biosynthesis wikipedia , lookup

Biochemistry wikipedia , lookup

Transcript
BIOCHEMISTRY
1.
Which of the following single-base DNA gene coding changes is least
likely to significantly affect the function of resulting protein? (Harper pages
361-363)
A.
B.
C.
D.
2.
Which of the following is a property of enhancer elements? (Harper pages
384-385)
A.
B.
C.
D.
3.
Saturated fatty acids increase fluidity.
Cholesterol is an amphipathic molecule.
The inner and outer sheets are symmetrical to each other.
They contain transport proteins, which are mainly peripheral.
Which of the following defines the specificity of hormone activity in a cell?
(Harper page 435)
A.
B.
C.
D.
6.
Chromatin remodelling
Alternative splicing
RNA editing
Capping of the mRNA
Which of the following is true of biological membrane structure? (Harper
pages 415-421)
A.
B.
C.
D.
5.
They do not work located downstream of the promoter.
They could work when located long distances from the promoter.
Upon activation, they could repress transcriptional rate.
They are proteins binding to DNA sequences.
Which of the following is considered as a transcriptional regulation
control? (Harper pages 383)
A.
B.
C.
D.
4.
Missence mutation resulting to histidine-to-glutamic acid change
Silent mutations in the third codon base
Frameshift mutation
Nonsense mutation
The synthesis of the hormone in the target cell
The presence of the hormone receptor
The upregulation of the hormone receptor in a cell
Absence of other target cells adjacent to hormone-producing cells
Which of the following is true of hormones? (Harper pages 437-38)
A. Hormones with intracellular receptors are usually hydrophilic.
B. Lipophilic hormones need transport proteins.
C. Cathecholamines and steroid hormones act via a plasma membrane
receptor.
D. Steroid hormones are derivatives of amino acids.
1
7.
Which vitamin is essential for one-carbon metabolism? (Harper page 494)
A.
B.
C.
D.
8.
Thiamine
Riboflavin
Niacine
Folic acid
Which of the following correctly characterize a component of the
extracellular matrix? (Harper 535-541)
A. Collagen type IV, a component of the basement membrane, form
compact bundles.
B. Elastin is essential for cell-cell communication.
C. Fibronectin is involved mainly in cell adhesion and signaling.
D. Laminin acts intracellularly as a major component of the cytoskeleton.
9.
Which of the following is NOT correctly paired? (Harper page 593)
A.
B.
C.
D.
10.
IgA: found in secretions
IgM: transplacental passage
IgG: opsonization
IgE: allergic responses
Which of the following statements characterizes ATP? (Harper page 84)
A.
B.
C.
D.
It is used in the synthesis of DNA.
It contains 3 phosphate bonds for energy storage.
It is used to drive thermodynamically unfavorable reactions.
It is produced mainly by substrate level phosphorylation.
11.
Enzymes, as biological catalysts accelerate velocity of reactions by:
A. lowering the free energy of activation of reactions
B. increasing transition state of reactants and products
C. lowering the net energy change between initial and final states of the
reaction
D. increasing the equilibrium constant (Keq) of the reaction
12.
A general class of enzymes that join molecules by creating a new
chemical bond at the expense of ATP hydrolysis
A. lyase
B. isomerase
C. ligase
D. hydrolase
13.
An enzyme catalyzing the reaction: lecithin + cholesterol -- fatty acid
cholesterol ester + lysolecithin is classified as:
A. oxidoreductase
B. lipase
C. transferase
D. hydrolase
14.
The Michaelis Menten constant is equal to substrate concentration when:
initial velocity is ½ Vmax
Vmax is doubled
Km= ½ substrate concentration
Km/Vmax = 1
2
15.
The kinetic parameter that reflects the affinity of the enzyme for its
substrate is called:
A. Vmax
B. Q10
C. Km
D. Kcat
16.
Double reciprocal plot of the Michaelis Menten equation is called:
A. Hanes-Wolf
B. Eadie Hofstee
C. Lineweaver Burk
D. Briggs-Haldane
17.
In the Lineweaver Burk plot, the slope of the line is equal to:
A. -1/Km
B. Km/Vmax
C. 1/Vmax
D. 1/S
18.
Which statement is true about competitive inhibition?
A. Inhibitor binds to the allosteric site of the enzyme
B. Km is constant while Vmax decreases
C. Usually the inhibitor structurally resembles the normal substrate
D. Both the Km and Vmax decrease
19.
Serum enzymes are elevated following tissue injury and death due to:
A. enzyme induction secondary to stimulation of protein synthesizing
machinery of the cell
B. allosteric enzyme activation by positive activators
C. increased cell membrane permeability and leakage of enzymes
into blood
D. proteolytic activation of proenzymes or zymogens
20.
Which of the following coenzymes is not appropriately paired with the
enzymatic reaction it is associated with?
E. tetrahydrofolate- transamination
F. biocytin-carboxylation
G. NAD- dehydrogenation
H. Coenzyme A- transacylation
21.
The active form of Vitamin B12 coenzyme:
A. 5’-deoxyadenosylcobalamin
B. hydrocobalamin
C. aquocobalamin
D. cobamide
22.
In Bioenergetics, the amount of energy available for the performance of
useful work is termed:
A. entropy
B. free energy
C. enthalpy
D. equilibrium energy
3
23.
During biological oxidations, reduced coenzymes such as NADH and
FADH2 generate energy when oxidized thru the:
A. Krebs Cycle
B. Electron Transport Chain
C. Cytochrome P450 system
D. Embden Meyerhof Pathway
24.
The final acceptor of electrons in the Respiratory Chain is:
A. cytochrome oxidase
B. coenzyme Q
C. iron sulfur center
D. oxygen
25.
Substrate level phosphorylation differs from oxidative phosphorylation in
that:
A. oxidative phosphorylation synthesizes ATP via the Electron
Transport Chain
B. substrate level phosphorylation only takes place in the cytosol
C. oxidative phosphorylation is coupled to a direct transfer of high energy
~Pi from a high energy compound to ADP
D. Substrate level phosphorylation is more energy efficient per molecule
of substrate oxidized
26.
Which of the following affects biological oxidation by dissociating
mitochondrial respiration from phosphorylation?
A. 2,4 dinitrophenol
B. B. rotenone
C. C. antimycin
D. cyanide
27.
Which of the following statements is correct regarding digestion of
carbohydrates?
A. Digestion of dietary polysaccharides takes place to a significant extent
in the stomach due to acid hydrolysis.
B. Digestion initially takes place in the mouth through the action of
- amylase.
C.
-1,4 glycosidic linkages in cellulose.
D. The final end products of digestion are all glucose molecules from
dietary carbohydrates
28.
In man, glucose from dietary cellulose could not be available for energy
because:
A. ce
B. human diet does not have a significant amount of cellulose
C. man inherently lacks enzyme to hydrolyze cellulose
D. cellulose is not made up of glucose units
29.
A 3 month old infant develops diarrhea and abdominal discomfort a
following a milk formula. However feeding history revealed that he was
able to tolerate breast feeding and the same milk formula at birth. The
most likely condition is
A.
- galactosidase deficiency
B. Milk allergy
C. Lactose malabsorption
D. Secondary lactose intolerance
4
30.
The major pathway of glucose metabolism that exists in all cells is the:
A. Pentose Phosphate Pathway
B. Citric Acid Cycle
C. Uronic Acid Pathway
D. Embden Meyerhof Pathway
31.
Hexokinase differs from glucokinase in which aspect?
A. Glukokinase has a higher affinity to glucose
B. Hexokinase is induced by insulin
C. Glucokinase phosphorylates glucose to glucose 6 phosphate
D. Hexokinase is inhibited by glucose 6 phosphate
32.
The end product of glycolysis in exercising muscle and red blood cells is:
A. pyruvic acid
B. acetic acid
C. lactic acid
D. acetoacetic acid
33.
The most important positive regulator of the committed step of glycolysis
is the availability of
A. fructose 1,6 bisphosphate
B. ATP
C. Citrate
D. fructose 2,6 bisphosphate
34.
True of the The Malate Aspartate Shuttle
A. can be considered as an anaplerotic reaction
B. less energetically efficient than the
C. transports NADH from the cytosol into the mitochondria for
oxidative phosphorylation
D. provides reducing equivalents for reductive biosynthesis of fats
35.
The initial step in the TCA Cycle is catalyzed by:
A. pyruvate dehydrogenase
B. lactate dehydrogenase
C. citrate synthase
D. -keto-glutarate dehydrogenase
36.
The Hexose Monophosphate Shunt is metabolically significant to nucleic
acid metabolism because it:
A. provides NADPH
B. furnishes pentose phosphates
C. is an alternative pathway for glycolysis
D. stabilizes structure of DNA
37.
The rate limiting step in the Pentose Phosphate Pathway is catalyzed by:
A. hexokinase
B. glucose-6 phosphate dehydrogenase
C. 6- phosphogluconolactonase
D. transketolase
38.
Which of the following enzymatic reaction is common to both glycolysis
and gluconeogenesis?
A. hexokinase
B. pyruvate carboxylase
C. phosphofructokinase 1
D. phosphotriose isomerase
5
39.
The inherent absence of this enzyme prevents muscle glycogenolysis to
maintain glucose homeostasis:
A. glycogen phosphorylase
B. glucose- 1- phosphatase
C. glucose-6-phosphatase
D. UDP-Glucose pyrophosphorylase
40.
Glycogen synthesis is activated while glycogenolysis is inhibited when:
A. glucagon/ insulin ratio is elevated
B. glycogen synthase is covalently modified by phosphorylation
C. cyclic AMP dependent protein kinase is activated
D. glycogen synthase I predominates over glycogen synthase D
41.
Mannose and Fructose enter the mainstream of carbohydrate metabolism
via:
A. Citric Acid Cycle
B. Embden Meyerhoff Pathway
C. Hexose Monophosphate Shunt
D. Glyoxylic Acid Cycle
42.
Fatty acids enter the mitochondrial matrix for oxidation in what form?
A. acyl-CoAs through active transport
B. as free fatty acids
C. following conversion to carnitine
D. complexed with carnitine as acyl-carnitine
43.
The number of reduced electron carriers generated when one mole of
palmitate is subjected to oxidation:
A. 2 FADH2 and 3 NADH + H+
B. 3 FADH2 and 2 NADH + H+
C. 7 FADH2 and 7 NADH + H+
D. 5 FADH2 and 5 NADH + H+
44.
Majority of the reducing equivalents necessary for fatty acid biosynthesis
are generated during conversion of:
A. 1,3 bisphosphoglycerate to 3 phosphoglycerate
B. pyruvate to malate
C. glucose 6-phosphate to 6 phosphogluconate
D. 6 phosphogluconate to ribulose 5-phosphate
45.
How many moles of NADPH + H+ can be derived from the pathway utilized
in the transport of acetyl CoA out of the mitochondria?
A. 8 moles
B. 10 moles
C. 12 moles
D. 14 moles
46.
The site of action of the NSAIDS is:
A. thromboxan synthase
B. prostaglandin cyclooxygenase
C. lipoxygenase
D. prostacyclin synthase
6
47.
Hepatocytes contribute to the overall body content of ketone bodies
primarily because of the absence of:
A. β-ketothiolase necessary to hydrolyze acetoacetyl CoA
B. HMG-CoA lyase
C. HMG-CoA synthetase
D. 3-ketoacyl-CoA transferase or acetoacetate succinyl CoA
transferase
48.
The principal plasma carrier of dietary triacylglycerols is:
A. serum albumin
B. LDLs
C. HDLs
D. chylomicrons
49.
Activation of lipoprotein lipase requires which apoprotein?
A. Apo E
B. Apo B-100
C. Apo C-II
D. Apo B-48
50.
VLDLs are synthesized in the liver for transport of which of the following to
the peripheral tissues?
A. triacylglycerols
B. apoproteins for HDL synthesis
C. LDL receptors
D. apo B-100
51.
The committed step of cholesterol biosynthesis is catalyzed by this
enzyme:
A. squalene synthase
B. HMG CoA reductase
C. mevalonate kinse
D. HMG CoA lyase
52.
High dietary intake of cholesterol leads to:
A. increased HMG CoA reductase synthesis
B. decreased apo B-100 synthesis
C. decreased ACAT activity
D. decreased LDL receptor synthesis
53.
Several of the enzymes of cholesterol biosynthesis require which of the
following cofactors?
A. biotin
B. NADPH
C. FADH2
D. TPP
54.
The molecule that helps stabilize the action of pancreatic lipase on dietary
lipids:
A. colipase
B. chenodeoxycholic acid
C. cholic acid
D. enteropeptidase
7
55.
Source of the nitrogen atoms required for animal metabolism can be
directly obtained from:
A. atmospheric nitrogen
B. NO2
C. ammonia
D. nitric oxide
56.
Aside from urea, the main circulatory system carrier of nitrogen atoms
destined for excretion by the kidney is:
A. glutamate
B. asparagine
C. alanine
D. glutamine
57.
A deficiency in the abilty to secrete enterokinase leads to which of the
following?
A. abnormally low levels of intestinal zymogens
B. abnormally low levels of active intestinal proteases
C. abnormally low levels of undigested proteins in lower GIT
D. increase plasma levels of serum amylase
58.
Transamidation reaction include which of the following?
A. formation of liver glutamate from α-ketoglutarate
B. formation of liver glutamine from glutamate
C. formation of liver asparagine from aspartate
D. formation of liver oxaloacetate from alanine
59.
The important cofactor for the transaminases:
A. pyridoxal phosphate
B. tetrahydrofolate
C. biocytin
D. tetrahydrobiopterin
60.
The production of alanine by peripheral tissue, especially by the muscle, is
primarily dependent on:
A. allosteric effectors of ALT
B. intracellular levels of pyruvate
C. intracellular levels of glutamate
D. intracellular levels of NADH
61.
Nitrogen of muscle alanine can be derived from tissue ammonia through
this enzyme action:
A. glutamate dehydrogenase
B. alanine dehydrogenase
C. glutamine synthetase
D. alanine synthetase
62.
Sulfur for cysteine synthesis is derived from :
A. sulfites
B. H2S
C. Coenzyme A
D. methionine
63.
Severe PKU and shortened life span is a result of:
A. excess dietary phenylalanine
B. excess dietary tyrosine
C. deficiency of dihydrobiopterin reductase
8
D. deficiency of tetrahydrofolate
64.
Serine is synthesized by:
A. transamination of an amino acid with α-ketoglutarate as an amino
acceptor
B. transamination of an α-ketoacid derived from TCA
C. transamination of an α-ketoacid derived from EMP
D. modification of an essential amino acid
65.
Lysine is an essential amino acid because:
A. animals cannot make the lysine carbon skeleton
B. the ε-amino group of lysine cannot be formed by an
aminotransferase
C. the α-carboxylic cannot be added to urocanate
D. lysine cannot be subjected to deamidation
66.
Amino acids that are strictly ketogenic:
A. lysine and leucine
B. leucine and isoleucine
C. valine and isoleucine
D. tyrosine and tryptophan
67.
Choose the correct precursor-product relationship from the following:
A. tyrosine is a precursor to dihydroxyphenylalanine
B. tyrosine is a precursor to melatonin
C. melatonin is a precursor of serotonin
D. dopamine is a precursor to norepinephrine
68.
The Watson-Crick model or the B-form of DNA is typified by:
A. 12 base pairs per helical turn
B. a smaller number of purines than pyrimidines
C. bases located in the interior of the right-handed helix
D. two single strands that run in the 5’ 3’ direction
69.
Guanine-cytosine (G-C) base pairs in a DNA molecule are more stable to
denaturation than adenine-thymine (A-T) pairs because:
A. adenine and thymine have bulkier structures than guanine and
cytosine
B. A-T base pairs exhibit less electron clouds around them
C. G-C base pairs contain more hydrogen bonds than A-T pairs
D. the phosphodiester bonds among G-C base pairs are weaker
70.
DNA is distinguished from RNA by:
A. a high content of modified bases
B. its component sugar, ribose
C. its single stranded structure
D. the presence of thymine instead of uracil
71.
Anticancer drugs like 5-fluorouracil and 6-mercaptopurine act against
malignant cells by:
A. being incorporated into DNA before replication
B. causing the denaturation of DNA in cancer cells
C. inactivating protein synthesis
D. inhibiting the enzymes that synthesize thymidylic acid
9
72.
Which of the following is a nucleotide?
A. uracil
B. cytidine
C. guanosine
D. adenylic acid
73.
Which vitamin is essential in the synthesis of purine nucleotides?
A. ascorbic acid
B. folic acid
C. niacin
D. thiamin
74.
Which clinical condition does not involve a defect in purine metabolism?
A. adenosine deaminase deficiency
B. gout
C. Lesch-Nyhan syndrome
D. orotic aciduria
75.
Salvage reactions that generate purine and pyrimidine nucleosides and
nucleotides are characterized as:
A. involving the same enzymes as in de novo synthesis
B. more efficient than nucleotide synthesis from new precursors
C. providing greater output of purines and pyrimidines
D. requiring less energy than does the de novo synthesis
76.
The enzyme that catalyzes the committed step of purine nucleotide de
novo pathway:
A. carbamoyl phosphate synthetase I
B. glutamine amidophosphoribosyl transferase
C. carbamoyl phosphate synthetase II
D. aspartate transcarbamoylase
77.
In the management of gout, allopurinol is given to inhibit xanthine oxidase,
the enzyme that catalyzes the synthesis of:
A. allantoate
B. ammonia
C. urea
D. uric acid
78.
The correct flow of genetic information according to the Central Dogma of
Molecular Biology is:
A. replication -----> transcription ----> translation
B. replication -----> translation ----> transcription
C. transcription ---- > replication ----> transcription
D. translation ----> replication ----> transcription
79.
Transfer RNA (tRNA) as an adaptor molecule serves to:
A. carry the nucleotide sequences that will dictate the amino acid to be
synthesized
B. hold the promoter site that will be recognized by the polymerase
C. recognize the triplet codon in mRNA via its anti-codon arm
D. synthesize the amino acid to be incorporated into the peptide
10
80.
Degeneracy of the Genetic Code means that:
A. an amino acid can only be decoded by a single codon
B. a single codon encodes only one amino acid
C. multiple codons may encode the same amino acid
D. with time, codons degenerate into senseless code
81.
The final product of gene expression is:
A. amino acid
B. nucleotide
C. polynucleotide
D. protein
82.
Normal growth and development of cells are controlled by genes known
as:
A. leader genes
B. non-coding genes
C. proto-oncogenes
D. transforming genes
83.
The type of mutation that involves the substitution of a purine base with
another purine base is:
A. transition
B. transposition
C. transpyrimidation
D. transversion
84.
This mineral helps guard against DNA damage as a cofactor of
glutathione:
A. iron
B. manganese
C. selenium
D. zinc
85.
A tool in molecular biology in which a DNA sequence of interest is
exponentially amplified in vitro by repeated cycles of enzymatic DNA
synthesis is:
A. DNA sequencing
B. in situ hybridization
C. Northern blotting
D. polymerase chain reaction
86.
In the construction of recombinant DNA, which enzyme is used to cut DNA
at precise base sequences?
A. DNA polymerase
B. DNA-RNA exonuclease
C. restriction endonuclease
D. T4 DNA ligase
87.
In recombinant DNA technology, antibiotic resistance genes in plasmids
are used to:
A. control copy number in bacteria
B. facilitate transformation of host cells
C. provide a marker for start of replication
D. select and screen for recombinant DNA clones
11
88.
The pain felt by a patient with an enlarging malignant tumor is partly
contributed by the:
A. exaggerated metabolism of nutrients by the tumor cells
B. loss of enzymes for glucose oxidation in cancer tissue
C. release of cytokines such as IL-1 and IL-6 by the tumor cells
D. overgrowth of blood vessels in the interior of the cancer mass
89.
Malignancy may be naturally prevented if the mutated cancer gene was
repaired:
A. after gene expression
B. before DNA replication
C. following transcription
D. shortly after DNA synthesis
90.
With persistence of a mutation in cells during organogenesis, the resulting
event is:
A. development of cancer
B. inactivation of genetic disorders
C. occurrence of physical malformations
D. promotion of sterility
91.
To specifically identify the mutation in the affected DNA segment in a
patient with cancer, which blot transfer method should be used?
A. Northern
B. Southern
C. Southwestern
D. Western
92.
The enzyme, Taq polymerase is used in Polymerase Chain Reaction
(PCR) because it:
A. facilitates annealing of primers
B. is resistant to heat denaturation
C. promotes DNA polymerization
D. provides optimum condition for DNA synthesis
93.
A 60-year-old woman has developed red, roughened skin in sun-exposed
areas over the past 2 years. She also has a chronic, watery diarrhea. On
physical examination she exhibits memory loss with confusion. These
findings are most consistent with which of the following vitamin
deficiencies?
A. Thiamine
B. Pyridoxine
C. Niacin
D. Vitamin A
94.
A clinical study is performed involving dietary iron metabolism in adults. It
is observed that intestinal absorption of iron can be enhanced in patients
with iron deficiency anemia by supplementing their diet with another
nutrient. Which of the following vitamins is most likely to have this effect?
A. D
B. E
C. C
D. B3
12
95.
A 6-year-old child was born at term, with no congenital anomalies. She is
now only 70% of normal body weight, though she shows dependent
edema of the lower extremities as well as an enlarged abdomen. Her
flaking skin shows irregular areas of depigmentation, hyperpigmentation,
and desquamation. These findings are most suggestive of which of the
following nutritional problems?
A. Marasmus
B. Kwashiorkor
C. Niacin deficiency
D. Vitamin A Deficiency
96.
A 41-year-old man has had increasing dyspnea for the past year. On
physical examination he has diffuse crackles at lung bases. A chest x-ray
shows pulmonary edema and cardiomegaly. Echocardiography shows an
ejection fraction of 40%. Laboratory studies show hemoglobin 14 g/dL,
hematocrit 42%, and WBC count 8320/microliter. A deficiency in which of
the following vitamins is most likely to produce these findings?
A. A
B. B1
C. B2
D. K
97.
A 7-year-old child has complained of pain in his legs for the past year. On
physical examination, there is bowing deformity of his lower extremities.
Plain film radiographs of his lower legs shows widened epiphyses and
bowing of tibiae. Bone mineral density appears normal, consistent with
failure of osteoid matrix formation. Which of the following vitamin
deficiencies is this child most likely to have?
A. D
B. E
C. C
D. B3
98.
A 28-year-old man has a history of multiple and recurrent pulmonary
infections since childhood. He also has noted foul smelling stools for the
past 10 years. Laboratory studies show an elevated sweat chloride test.
He has a quantitative stool fat of 10 g/day. A deficiency state involving
which of the following nutrients is most likely to develop in this patient?
A. Vitamin B1
B. Vitamin D
C. Calcium
D. Vitamin A
99.
A clinical study is performed to compare the risk for health problems in
obese persons with a BMI OF 35, compared with a control group of
persons with a BMI between 20 and 25. Persons in these two groups are
followed for 20 years. Which of the following conditions is most likely to
appear equally in both groups?
A. Cholelithiasis
B. Cirrhosis
C. Hypertension
D. Diabetes mellitus
13
100.
A 15-year-old girl has been under a physician's care for the past year after
diagnosis of anorexia nervosa. Her BMI is now 18. On physical
examination she has cheilosis. Laboratory studies show hemoglobin 13.7
g/dL, hematocrit 41.0%, MCV 88 fL, platelet count 191,055/microliter, and
WBC count 4930/microliter. Her serum glucose is 66 mg/dL. Which of the
following nutrient deficiencies is most likely to cause this patient's
findings?
A. Riboflavin
B. Vitamin C
C. Folic Acid
D. Iron
101.
The metabolic function of the pentose phosphate pathway is to:
A. act as a source of ADP biosynthesis.
B. generate NADPH and pentoses for the biosynthesis of fatty acids
and nucleic acids.
C. participate in oxidation-reduction reactions during the formation of H2O.
D. provide intermediates for the citric acid cycle.
102.
The FNRI Philippine RENI Committee 2002 recommends the following
protein intake:
A. 1.14 g/kg DBW/day
B. 2.14 g/kg DBW/day
C. 3.14 g/kg DBW/day
D. 4.14 g/kg DBW/day
103.
Number of moles of ATP generated from the complete oxidation of
pyruvate as it undergoes the pyruvate dehydrogenase reaction and the
Kreb’s cycle is:
a. 12
c. 14
b. 13
d. 15
Page 113 Harper’s 26th Edition Chapter 16
104.
Which of the following enzymes when deficient wil result in the
accumulation of gangliosides in patients suffering for Tay Sach’s Disease?
a. glucocerebrosidase
c. sphingomyelinase
b. aryl sulfatase
d. hexosaminidase
th
Page 203 Harper’s 26 Edition Chapter 24
105.
Which of these is deficient in Cori’s disease?
a. glucose-6-phosphate
b. glycogen phosphorylase
Page 152 Harper’s 26th Edition Chapter 18
c. branching enzyme
d. debranching enzyme
106.
Just before breakfast, what is the source of glucose in the body?
a. glucose
c. ketones
b. glycogen
d. fatty acids
Page 102,107-108 Harper’s 26th Edition Chapter 13
107.
For every mole of glucose that goes through the glycolytic pathway, what
is the net number of pyruvate generated?
a. one
c. three
b. two
d. four
Page 136-144 Harper’s 26th Edition Chapter 17
14
108.
Which of these is the primaryfunction of the Pentose Phosphate Pathway?
a. generation ATP
b. NADPH generation
c. NADPH generation and production of ribose residues
d. Provision of ribose residues of nucleotide and nucleic acid
synthesis
Page 163-168 Harper’s 26th Edition Chapter 20
109.
Which enzyme generates NADPH + H needed to reduce GSSG:
a. glutathione reductase
c. aldehyde
dehyrogenase
b. glucose 6PO4 dehydrogenase
d. lactate dehydrogenase
th
Page 156-165 Harper’s 26 Edition Chapter 19
110.
Glucose 6 phosphate dehydrogense deficiency is characterized by which
one of the following?
a. accumulation of glucose -6-phosphate in the tissue
b. decreased formation of NADPH
c. increased synthesis of reduced glutathione
d. may manifest as hemorrhagic anemia
Page 169-170 Harper’s 26th Edition Chapter 20
111.
The Citric acid cycle as the final common pathway in fuel metabolism
provides greatest amount of energy by being directly linked to:
a. glycerophosphate shuttle
c. glycolysis
b. malate aspartate shuttle
d. electron transport chain
Page 130-133 Harper’s 26th Edition Chapter 16
112.
Cyanide poisoning is very fatal. This is bacuse cyanide has this effect of
the Electron Transport Chain:
a. inhibits complex III
b. inhibits cytochrome oxidase
c. uncouples oxidative phosphorylation
d. ienhance the transport of electrons
Page 92 Harper’s 26th Edition Chapter 12
113.
At high concentration of blood glucose, as in uncontrolled diabetes
mellitus, hemoglobin reacts with glucose to form this compound which can
be assayed to monitor glycemic control
a. methemoglobin
b. hemochrome
c. glycosylated hemoglobin
d. hemin
Page 47 Harper’s 26th Edition Chapter 6
114.
The precursor of the sugar-phosphate moieties in the de novo synthesis of
both purine and primidine nucleotide is:
a. glucose 1 phosphate
b. glucose 6 phosphate
c. ribose 1 phosphate
d. ribose 5 phosphate
Page 286-292 Harper’s 26th Edition Chapter 33
115.
The highest reservior of calories in muscle tissues is:
a. Fats
c. glycogen
b. proteins
d. ketone bodies
Page574-576 Harper’s 26th Edition Chapter 49
15
116.
cAMP exert its effect as a second messenger by:
a. inducing release of cytokines
b. activating Protein Kinase C
c. activating cyclic adenylase
d. activating Protein Kinase A
Page 457-468 Harper’s 26th Edition Chapter 43
117.
Deficiency in the intake of carbohydrates and fats can lead to:
a. increase in protein requirements
b. increase intake of calories
c. increase deposition of fats
d. decrease in the catabolism of body proteins
Page 478-479 Harper’s 26th Edition Chapter 44
118.
Gluconeogenesis is the major source blood sugar during:
a. the fed state
c. prolonged fasting
b. periods between meals
d. early refed state
Page 158-161 Harper’s 26th Edition Chapter 19
119.
Melatonin is a specialized product of tryptophan that is involved with
a. pigment formation
c. circadian rhythm
b. vasodilatation
d. thyroid gland secretion
th
Page 267 Harper’s 26 Edition Chapter 31
120.
The receptor involved in the G-protein coupled signal transduction
pathway:
a. tyrosine kinase receptor
c. β adrenergic receptor
b. guanylyl cyclase receptor
d. janus kinase receptor
Page 459,464-465 Harper’s 26th Edition Chapter 43
121.
Dinitrophenol (DNP) and other uncouplers which uncouple oxidation and
phosphorylation are useful weight losing agents. They prevent ATP
sysnthesis in the Electron Transport Chain by which of the following
mechanism?
a. Electron Transport is blocked
b. Protons build up in the intermembrane space
c. There is reduced availability of ADP
d. The electrochemical gradient is dissipated
Page 95-96 Harper’s 26th Edition Chapter 12
122.
The most active form of vitamin D is:
a. 1,25 dihydroxycholecalciferol
c. ergocalciferol
b. vitamin D3
d. 7- dehydrocalciferol
Page 484-486 Harper’s 26th Edition Chapter 45
123.
Plant fat is high in unsaturated fatty acids compared to animal fat. Which
one of the following characteristics BEST describes unsaturated fatty
acids in contrast saturated fatty acids?
a. lower rate of oxidation
b. lower rate of melting point
c. less water soluble
d. more solid at room temperature
Page 144 Harper’s 26th Edition Chapter 14
16
124.
This is the mechanism of action by which penicillin kills bacterial cells:
a. it inhibits bacterial DNA replication
b. it acts as an allosteric inhibitor to the enxzyme reverse
transcriptase
c. it is an analogue of D-phe-D-phe effectively preventing the
synthesis of tetrahydrofolates
d. it covalently binds to the active site of the enzyme for cell wall
synthesis
Page 371-372 Harper’s 26th Edition Chapter 38
125.
An attack of acute asthma has been attributed to leukotrienes. Which of
the following statements is TRUE regarding leukotrienes?
a. produced mainly in the nervous tissue
b. components of slow reacting substance of anaphylaxis
c. descrease vascular permeability
d. inhibit the release of lysosomal enzymes
Page 194-196 Harper’s 26th Edition Chapter 23
126.
This statement accurately describes Vitamin K:
a. Menaquinone-7 is the major form of Vitamin K found in plants
b. It is found only in green leafy vegetables while intestinal bacteria
are not able to synthesize it
c. It functions as a cofactor to a carboxylase that acts on
glutamate residues of clotting factor precursors proteins
d. It main function is to protect the erythrocytes
Page 486 Harper’s 26th Edition Chapter 45
127.
Bilirubin is an important endproduct during the course of hemolysis. Which
of the following statements is TRUE regarding bilirubin?
a. excreted from the liver in the form of bilirubin diglucuronides
b. conjugation mainly it takes place in the kidneys
c. unconjugated bilirubin is readily excreted in the urine
d. unconjugated hyperbilirubinemia due to complete obstruction of the
common bile ducts is generally accompanied by increased
urobilinogen in the urine
Page 280-284 Harper’s 26th Edition Chapter 32
128.
Kwashiorkor occurs when there is:
a. lack of calories in the diet
b. lack of calories from protein in the diet
c. lack of carbohydrates and protein in the diet
d. lack of all the macro and micro nutrients in the diet
Page 237,478-479 Harper’s 26th Edition Chapter 28 , 44
129.
Cancer can be attributed to mutation in the DNA. A mutation that converts
an amino acid codon to a stop codon is:
a. silent
c. missense
b. frameshift
d. nonsense
Page 362 Harper’s 26th Edition Chapter 38
130.
The process by which a polypeptide’s destination in the cell is determined
by which of these?
a. Codons
c. signal peptides
b. molecular chaperones
d. signal
recognition particles
Page 498,503-504,508 Harper’s 26th Edition Chapter 46
17
131.
Which of the following histones is uniquely associated with the linker
DNA?
a. H3
c. H4
b. H1
d. H2A
th
Page 314-315 Harper’s 26 Edition Chapter 36
132.
Which of the following is a nucleoside?
a. guanylic acid
c. guanosine 5’-monophosphate
b. dUMP
d. cytidine
th
Page 286-287 Harper’s 26 Edition Chapter 33
133.
Which of the following about the salvage pathway of purine nucleotide
synthesis is correct?
a. Deficiency of hypoxanthine-guanine phosphoribosyl
transferase is the cause of Lesch-Nyan syndrome
b. The rate limiting enzyme is PRPP amidotransferase
c. The donor of the ribose 5-phosphate is the HMP shunt
d. The salvage pathway for the free bases require more ATP than the
novo sysnthesis of purine nucleotide
Page 294 Harper’s 26th Edition Chapter 34
134.
An individual who has fasted for more than 24 hours will have an
increased production of which amino acids in his tissues?
a. aspartate and glutamine
b. glutamate and asparagine
c. valine, isoleucine and leucine
d. glutamine and alanine
Page 14-20 Harper’s 26th Edition Chapter 3
135.
Which amino acid is purely ketogenic ?
a. leucine
c. isoleucine
b. tryptophan
d. proline
Page 15,259-260 Harper’s 26th Edition Chapter 3,30
136.
Kwashiorkor occurs when there is:
a. lack of calories in the diet
b. lack of calories from protein in the diet
c. lack of carbohydrates and protein in the diet
d. lack of all the macro and micro nutrients in the diet
Page 478-479 Harper’s 26th Edition Chapter 44
137.
This amino acid is the source of most of the blood glucose during
starvation:
a. glutamine
c. glutamate
b. alanine
d. aspartate
Page 236 Harper’s 26th Edition Chapter 27
138.
Allopurinol is used in gout bacause it is an allosteric inhibitor of xathine
oxidase which is responsible for synthesizing this excretory metabolite of
purines:
a. allanton
c. urea
b. uric acid
d. ammonia
Page 301 Harper’s 26th Edition Chapter 35
18
139.
The catecholamines, epinephrine and dopamine are synthesized from this
amino acid:
a. tyrosine
c. arginine
b. isoleucine
d. threonine
Page 445 Harper’s 26th Edition Chapter
140.
Which of the following is the most important role of amino acids in the
body?
a. source of energy
b. precursor of nonprotein nitrogen containing substances
c. building blocks of tissue protein
d. precursor glucose
Page 249 Harper’s 26th Edition Chapter 30
141.
Most of the ammonia that are used for the synthesis of urea are directly
derived from the deamination of:
a. leucine
c. isoleucine
b. aspartic acid
d. glutamic acid
Page 243-244 Harper’s 26th Edition Chapter 29
142.
A person suffering from gout should decrease his intake of:
a. bread
c. egg
b. banana
d. sardines
th
Page 229 Harper’s 26 Edition Chapter 26
143.
Most of the function membranes are accomplished by:
a. proteins
c. carbohydrates
b. phospholipids
d. cholesterol
th
Page 416-417 Harper’s 26 Edition Chapter 41
144.
Which statements is BEST describes the arrangement of proteins and
lipids in membranes?
a. Protein form a continous sheet on either side of the phospholipid
bilayer
b. Proteins occur in the lipid bilayer as discontinous particles
c. Protein molecules are sandwiched between the exoplasmic and
cytoplasmic lipid monolayers
d. Lipid molecules “float” on a sea proteins
Page 416-417 Harper’s 26th Edition Chapter 41
145.
Excessive ingestion of raw eggwhite is detrimental because it inhibits this
step in extramitochondrial lipogenesis:
a. formation of malonyl CoA
c. reduction step
b. binding of malonyl to acyl carrier protein d. deacylation reaction
Page 173-174 Harper’s 26th Edition Chapter 21
146.
Gelatin has a high collagen content. It is not a good source of protein
because:
a. collagen contains a limited variety of amino acids
b. collagen cannot be digested
c. too much collagen causes brittle bones
d. collagen contains more sugars than amino acids
Page 38 Harper’s 26th Edition Chapter 5
19
147.
Semi-conservative replication of DNA means:
a. DNA systhesis occurs one strand at a time
b. Each new DNA strand forms a pair with the template strands
c. The two new DNA strands pair with each other
d. Duplication occurs in opposite direction in two strands
Page 306-307 Harper’s 26th Edition Chapter 35
148.
Which of the following is the BEST describes the action of 5 FU in the
treatment of cancer?
a. inhibits synthesis of DNA
b. inhibits protein synthesis
c. depletes the cancer cells of necessary nutrients like vitamins
d. depletes the cancer cells of required oxygen
Page 328-330 Harper’s 26th Edition Chapter 36
149.
Which one of the following is true regarding RNA?
a. Messenger RNA contain peculiar bases
b. The codon is found in trasfer RNA
c. Ribosomal RNA contain the anticodon
d. Transfer RNA carries the amino acid
Page 306-312,341-357 Harper’s 26th Edition Chapter 35,37
150.
During DNA replication, which enzyme joins Okazaki fragments?
a. topoisomerase
c. DNA ligase
b. RNAase II
d. helicase
th
Page 328 Harper’s 26 Edition Chapter 36
151.
Degeneracy of the genetic code denotes the existence of:
a. multiple codons for a single amino acid
b. codons consisting only of 2 bases
c. base triplets that do not code for any amino acid
d. codons that include one or more of the “unusual” bases
Page 359 Harper’s 26th Edition Chapter 38
152.
In the normal resting state of humans, muscles will mainly use up energy
provided by:
a. carbohydrates
c. fats
b. branced chain amino acids
d. muscle glycogen
Page 574-576 Harper’s 26th Edition Chapter 49
153.
Familial hypercholesterolemia is due to
a. deficiency or defective LDL receptors
b. mutation on HMG-CoA reductase
c. increased dietary intake of cholesterol
d. decreased excretion of cholesterol
Page 228 Harper’s 26th Edition Chapter 26
154.
Which of the following is the most metabolically active plasma lipid?
a. nonesterified fatty acid
c. free cholesterol
b. lecithin
d. glucocerebroside
th
Page 57 Harper’s 26 Edition Chapter 7
155.
Deficiency of lipoprotein lipase will result in a increased plasma level of
which of the following?
a. chylomicron remnant
c. LDL
b. VLDL
d. HDL
Page 125,126,207-208 Harper’s 26th Edition Chapter 15,25
20
156.
Which of the following is needed for the transport of long-chain fatty acids
from the cytosol to the mitochondrial matrix?
a. carnitine
c. glutathione
b. S-adenosylmethionine
d. phosphoadenosine
phosphosulfate
Page 180-187 Harper’s 26th Edition Chapter 22
157.
Major enzyme that digest dietary triacylglycerol
a. lingual lipase
c. lipoprotein lipase
b. pancreatic lipase
d. hormone sensitive
lipase
Page 475-476 Harper’s 26th Edition Chapter 44
In the liver cholesterol is mainly disposed by:
a. being exported as HDL
b. being exported as VLDL
c. being secreted in the bile as free cholesterol
d. being converted to bile acids
Page 225-227 Harper’s 26th Edition Chapter 26
158.
159.
Which is the immediate source of energy of ATP synthesis?
a. electron transfer to O2
c. ADP and Pi
b. electrochemical gradient
d. solar energy
th
Page 96-97 Harper’s 26 Edition Chapter 12
160.
The link between the catabolic and anabolic reactions in the energy cycle
is:
a. ATP
c. H+
b. O2
d. NADH
Page 82-85 Harper’s 26th Edition Chapter 10
161.
Which of the following is a function of pyridoxal phosphate?
a. prosthetic group of transaminases
b. lowers plasma cholesterol
c. a coenzyme to many dehydrogenases
d. adds hydride ion to the pyridine ring
Page 50,491 Harper’s 26th Edition Chapter 7,45
162.
What is the role of oxygen in cellular respiration?
a. source of electrons to be passed along the respiratory chain
b. final receptor of electrons in the respiratory chain
c. transporter of electrons from cytosol to mitochondrial matrix
d. cofactor of ATP synthase
Page 92-101 Harper’s 26th Edition Chapter 12
163.
Which vitamin is solely synthesized by bacteria and is thus absent from all
plants?
a. Biotin
c. Folic acid
b. Ascorbic acid
d. Riboflavin
Page 494-495 Harper’s 26th Edition Chapter 45
164.
This vitamin is required for the synthesis of neurotransmitters like
epinephrine:
a. Niacin
c. Pyridoxine
b. Ascorbic acid
d. Riboflavin
Page495-496 Harper’s 26th Edition Chapter 45
21
165.
Deficiency of this mineral is prevalent among pregnant women living in
mountainous areas:
a. iron
c. copper
b. iodine
d. magnesium
Page 447-449 Harper’s 26th Edition Chapter 42
166.
Which of the ff. accumulates in carnitine deficiency in humans ?
A. glycogen
B. odd-numbered fatty acids *
C. triglycerides
D. sphingolipids
167.
Which regulatory action involves a reversible covalent modification of an
enzyme?
A. allosteric modulation
B. competitive inhibition
C. association of apoenzyme with a cofactor
D. phosphorylation of Ser hydroxyl on the enzyme *
168.
The Ping-pong mechanism of enzyme kinetics applies to which of the
reactions?
A. one-substrate
B. bisubstrate *
C. multiple substrate
D. hydrolytic
169.
Enzymes hasten the rate of chemical reactions through which of the ff. ?
A. covalent modification of coenzymes and reactants
B. enhancing proximity and local concentration of substrates
C. decreasing the free energy of activation *
D. increasing the over-all free energy change of the reaction
170.
What is the role of oxygen in electron transport system ?
A. accepts electrons from FADH2
B. the final acceptor of electrons *
C. a mobile electron carrier
D. donates electron to water
171.
Which of the ff. BEST explains why oxidation of FADH2 produces less
ATP than NADH?
A. FADH2 enters ETS via complex II *
B. FADH2 transfers less electrons through ETS than NADH
C. Electrons from FADH2 are not transferred to oxygen
D. inner mitochondrial membrane is impermeable to FADH2
172.
Which of the ff. BEST applies to Isoelectric ph of a protein?
A. ph is physiologic
B. net charge is zero *
C. protonation is maximum
D. ionization is maximum
173.
What kind of a reaction is interconversion of aldose and ketose ?
A. dehydration
B. reduction
C. oxidation
D. isomerization *
22
174.
In the over-all scheme of human metabolism, which organ plays impt role?
A. brain
B. liver *
C. muscle
D. kidney
175.
Which of these enzymes is important in protein digestion ?
A. ptyalin
B. trypsin *
C. invertase
D. pancreatic amylase
176.
The beta pleated sheet of a protein is classified as which level of protein ?
A. primary
C. tertiary
B. secondary *
D. super secondary
177.
What is the cause for the lactic acidosis that develops in thiamine
deficiency?
A. conversion of pyruvate to acetyl-CoA is blocked *
B. conversipon of pyruvate to oxaloacetate is blocked
C. the transketolase reaction that transfers 3-carbon units is blocked
D. the transaldolase reaction has increased activity
178.
Which of the ff. statements BEST describes collagen?
A. left handed helix
B. 2 left handed alpha helices
C. double helical structure
D. triple helix *
179.
Which of these BEST explains the role of HCL in the stomach?
A. kills H. pylori organisms
B. denatures proteins *
C. ideal ph for amylase action
D. stimulates pancreatic juice sec’n
180.
Which of the ff. bonds stabilizes the primary structure of a protein?
A. hydrogen *
B. disulfide
C. ionic
D. peptide
181.
Which of the ff. are products of the action of salivary amylase on
glycogen?
A. sucrose
B. maltose *
C. galactose
D. fructose
182.
A zwiterion is a molecule that contains which of the following?
A. a cation and anion *
C. polar and non-polar groups
B. more than one cation
D. more than one anion
183.
Which of the ff. is most important source of glucose during early fasting?
A. liver triglyceride
C. muscle protein
B. hepatic glycogen *
D. muscle glycogen
23
184.
A young infant, who was nourished by synthetic formula, had sugar in
blood & urine. This compound gave positive reducing sugar test but
negative with glucose oxidase test. Which of the ff. compounds is MOST
likely to be present in infant’s blood & urine?
A. glucose
C. Sorbitol *
B. fructose
D. maltose
185.
In normal resting state of humans, muscles will mainly use up energy that
is provided by which of the ff.?
A. carbohydrates
C. fats
*
B. muscle glycogen
D. amino acids
186.
The anemia that develops in patients on anti-malarial therapy may be due
to deficiency of enzymes that belong to which metabolic pathway?
A. Embden-Mayerhoff
C. hexose monophosphate *
B. Kreb’s TCA
D. Kreb’s-Hanseleit
187.
Which of the ff is the primary function of the Pentose Phosphate pathway?
A. ATP production
B. NADPH generation
C. NADPH generation & production of ribose units *
D. Provision of ribose units for nucleotide/ nucleic acid synthesis
188.
Which of the ff, is the major metabolic substrate required by RBC?
A. Glycerol
C. fructose
B. free fatty acids
D. glucose *
189.
Which of the ff. is true of the TATA box?
A. binds to anti-codon
C. encodes repressor proteins
B. binds RNA polymerase *
D. located in first structural gene
190.
SRS-A, the slow reacting substance of anaphylaxis, is composed of which
of the ff?
A. thromboxanes
C. leukotrienes *
B. prostaglandins
D. prostacyclins
191.
The regulation of cholesterol synthesis primarily occurs at which
enzymatic site?
A. 3-hydroxy-3-methylglutaryl CoA synthase
B.3-hydroxy-3-methylglutaryl CoA lyase
C.3-hydroxy-3-methylglutaryl CoA reductase *
D.3-methyl-3-hydroxylglutaryl CoA oxidase
192.
What is the major source of reducing equivalents used for fatty acid syn?
A. pentose phosphate pathway *
B. Embden-Mayerhoff pathway
C. the tricarboxylic acid cycle
D. Pyruvate dehydrogenase pathway
193.
What is the role of aspirin in the treatment of rheumatoid arthritis?
A. inhibits the release of arachidonic acid
B. directly neutralizes prostaglandins & thromboxanes
C. inactivates cyclooxygenase *
D. shuts down the synthesis of all eicosanoid
24
194.
Patients suffering from familial defect in Apo B 100 have significantly
elevated levels of plasma LDL’s. Which is the major reason for this?
A. inability of LDL to interact with the LDL receptor *
B. inability of LDL to activate lipoprotein lipase
C. inability of LDL to activate cholesterol transfer protein
D. LDL interaction with LDL receptors fail to stimulate endocytosis
195.
What is the pharmacological benefit of the drug – Lovastatin ?
A. inhibits intestinal absorption of dietary cholesterol
B. inhibits hepatic production of VLDL
C. inhibits HMG-CoA reductase *
D. absorbs bile acids, increasing their rate of elimination in the feces
196.
In an average adult, which % of body protein is turned over on a daily
basis?
A. 1 – 2 % *
C. 5 – 6
B. 3 – 4 %
D. 7 – 8 %
197.
Which vitamin combines with opsin to form the visual pigment in the eye ?
A. A *
C. E
B. D
D. K
198.
Which of the following is the cause of muscle fatigue following vigorous
exercise?
A. sodium and potassium imbalance
B. the utilization of ATP
C. accumulation of ADP and PI
D. accumulation of lactate *
199.
Which intermediate of pyrimidine biosynthesis is common to the urea
cycle?
A. ornithine
C. carbamoyl phosphate *
B. citrulline
D. carbamoyl aspartate
200.
Which of these is a function of nonsense codons?
A. they code for non-essential amino acids
B. they are responsible for degeneracy of the genetic code
C. they base pair with the anticodons
D. they may be used in the cell as termination signals *
201.
Which phase of protein synthesis does the release of newly synthesized
polypeptide from the ribosome take place?
A. initiation
C. release
B. elongation
D. termination *
202.
Which of the ff. directs the polypeptide’s ultimate destination in the cell ?
A. codons
C. signal peptides *
B. molecular chaperones
D. signal recognition particles
203.
What is the primary action of methotrexate, a chemotherapeutic agent ?
A. displaces cells of folate *
C. inhibits phosphorylation of free bases
B. inhibits cellular kinase activity D. inhibits synthesis of nucleotides
204.
Which of these is the first reaction in the metabolism of all sugars ?
A. hydrolysis
C. isomerization
B. proteolysis
D. phosphorylation *
25
205.
Which site on the operon is the site of RNA polymerase binding ?
A. operator
C. promoter *
B. regulator
D. structural
206.
What is the cellular site of synthesis of messenger RNA ?
A. nucleus
C. mitochondria
B. cytosol
D. Ribosome *
207.
Which of the following BEST applies to transfer RNA ?
A. largest of RNA specie
C. exists in at least 20 diff forms *
B. has no secondary structure
D. assoc. w/ histones in cytosol
208.
Which of the ff. is the biological function of DNA synthesis ?
A. production of normal RNA’s
C. prevents mutation
B. production of normal proteins D. assures viability of the specie *
209.
During RNA synthesis, DNA template sequence 5’ TpApGpCp 3’ would be
transcribed to produce which of the ff. sequences ?
A. 5’- TpApCpGp- 3’
C. 3’- ApUpCpGp-5’
B. 3’- GpCpTpAp- 5’
D. 5’- GpCpUpAp-3’ *
210.
Which of the ff. refer to sequential order of nucleotides in a polypeptide?
A. primary structure *
C. tertiary structure
B. secondary structure
D. quaternary structure
211.
Which of the ff. enzymes in replication functions as proof-reading device?
A. polymerase enzyme
C. 5’ exonuclease
B. 3’ exonuclease *
D. nuclease
212.
Which of these constitute the second level of organization of DNA?
A. nucleosome *
C. double helix DNA
B. chromosome
D. condensed metaphase chromosome
213.
Which of the ff. BEST describes Okazaki fragments?
A. fragments of DNA attached to an initiator component
B. fragments of continuous DNA which eventually are joined to form DNA
C. fragments of DNA that are copied in direction away from replic’n
fork *
D. synthesized fr. 5’ to 3’ & proceeds in same direction as replication fork
214.
The antimetabolite, 5-FU inhibits DNA replication by which mechanism?
A. intercalates into the DNA
B. cross-links adjacent thymines in the DNA
C. inhibts production of deoxythymidine monophosphate *
D. cross-links adjacent guanine residues in the DNA
215.
Which posttranscriptional processing prevents 5’ exonuclease attack?
A. splicing reactions
B. B.addition of extra COOH groups
C. poly A tail *
D. 7 methylguanosine triPO4
216.
Which of the ff. reactions does RNA-directed DNA polymerase catalyze?
A. synthesis of DNA complimentary to the RNA strand *
B.synthesis of DNA complimentary to template DNA strand
C. removal of wrong nucleotide inserted during DNA synthesis
D.synthesis of an RNA that is complimentary to RNA template
26
217.
During protein synthesis, which reads information in mRNA & transfers AA
to a growing polypeptide chain?
A. messenger RNA
C. heterogenous nuclear RNA
B. ribosomal RNA
D. transfer RNA *
218.
What is the cause of polyuria in diabetes mellitus ?
A. overdrinking of water because of polydipsia
B. decreased ADH activity
C. osmotic diuresis *
D. decreased aldosterone activity
219.
Which is TRUE about methemoglobinemias?
A. There is increased activity of methemoglobin reductase.
B. It can result as a side-effect of sulfonamides.
C. Increased reduction of Fe3+ of methemoglobin to Fe2+ occurs.
D. There is absence of the Bohr effect.
Reference: Murray, R.K., et al., Harper’s Biochemistry, 27th Ed., pp. 46-47.
220. One of the following is a saturated fatty acid:
A. Oleic acid
B. Linolenic acid
C. Palmitic acid
D. Arachidonic acid
Reference: Murray, R.K., et al., Harper’s Biochemistry, 27th Ed., p. 123.
221.
One of the following statements is true of the structure
CH3(CH2)4CH=CHCH2CH=CH(CH2)7COOH:
A. It is a saturated fatty acid with 18 C atoms.
B. Its common name is linolenic acid.
C. It has two double bonds located at C 9 and C 12.
D. Its other name is 18:3c9,12.
Reference: Murray, R.K., et al., Harper’s Biochemistry, 27th Ed., p. 122.
Lipid Metabolism
222. Which of the following is TRUE of fatty acid synthase enzyme complex?
A. It can synthesize two 18 carbon atom fatty acids simultaneously.
B. It is composed of two polypeptide chains but each monomer is an active
enzyme.
C. FAS enzyme complex has seven sub-units with catalytic activities.
D. The monomers are arranged in a head – to – tail configuration.
Reference: Murray, R.K., et al., Harper’s Biochemistry, 27th Ed., pp. 196197.
223.
The phosphopanthetheine side chain is part of which sub-unit of the fatty
acid synthase enzyme complex ?
A. acetyl transacylase
C. acyl carrier protein
B. ketoacyl synthase
D. malonyl transcylase
Reference: Murray, R.K., et al., Harper’s Biochemistry, 27th Ed., p. 196.
224.
Deficiency of this phospholipid results to Respiratory Distress Syndrome
of the newborn:
A. phosphatidylinositol
C. phosphatidylserine
B. phosphatidylinositide
D. dipalmitoyl-lecithin
Reference: Murray, R.K., et al., Harper’s Biochemistry, 27th Ed., p. 125.
27
The committed step for the β – oxidation of fatty acids is inhibited by
increased concentration of:
A. acetyl-CoA
C. malonyl-CoA
B. carnitine
D. palmitoyl – CoA
Reference: Murray, R.K., et al., Harper’s Biochemistry, 27th Ed., p. 195.
225.
226.
The complete ATP yield of a saturated C16 fatty acid is:
A. 106
C. 110
B. 108
D. 112
Reference: Murray, R.K., et al., Harper’s Biochemistry, 27th Ed., p.189.
227.
Which condition accelerates ketogenesis?
A. high carbohydrate diet
B. Increased [NAD+]/[NADH] ratio
C. low [beta-hydroxybutyrate]
D. decreased [Insulin]/[glucagon] ratio
Reference: Murray, R.K., et al., Harper’s Biochemistry, 27th Ed., pp. 192193.
228.
Ketone bodies can be utilized by extra-hepatic tissues because of this
enzyme:
A. beta-hydroxybutyrate dehydrogenase C. HMG-CoA lyase
B. HMG-CoA synthase
D. succinyl CoA acetoacetate
CoA
transferase
Reference: Murray, R.K., et al., Harper’s Biochemistry, 27th Ed., p. 191.
229. Accelerated ketogenesis occurs in which situation?
A. activation of CPT I
C.  glucagon/Insulin
B.  malonyl CoA
D. activation of acetyl CoA
carboxylase
Murray, R.K., et. al., Harper's Illustrated Biochemistry, 27th ed., p.193
230.
This disease results from the accumulation of phytanic acid in the brain
and kidneys of patients:
A. Tay- Sach’s disease
C. Refsum’s disease
B. Zellweger’s disease
D. Niemann Pick’s disease
Reference: Murray, R.K., et al., Harper’s Biochemistry, 27th Ed., p. 194.
231.
Which is TRUE regarding apoproteins?
A. Apo B 46 and B100 are synthesized in the liver.
B. Apo A, B, C and D are peripheral apoproteins.
C. Apo D is present exclusively in HDL.
D. Apo A II and C III are stimulate endothelial lipoprotein lipase.
Reference: Murray, R.K., et al., Harper’s Biochemistry, 27th Ed., pp. 218.
232.
The concentration of this lipoprotein is inversely related to the incidence of
coronary artery disease:
A. HDL1
C. HDL3
B. HDL2
D. pre β- HDL
Reference: Murray, R.K., et al., Harper’s Biochemistry, 27th Ed., p. 222.
233.
Which receptors are actively involved in reverse cholesterol transport?
A. LDL
C. LRP
B. ABCA 1
D. SR- B1
Reference: Murray, R.K., et al., Harper’s Biochemistry, 27th Ed., p. 222.
28
234. The only part of an odd-chain fatty acid that is glucogenic is:
A. acetyl CoA
C. propionyl CoA
B. butyryl CoA
D. succinyl CoA
Reference: Murray, R.K., et. al., Harper's Illustrated Biochemistry, 27 th ed.,
p.188
235. Which is the correct disease: enzyme deficiency pair?
A. Fabry's disease: Ceramidase
B. Gaucher's disease: Hexosaminidase A
C. Niemann - Pick disease: Sphingomyelinase
D. Tay-Sachs disease:  - glucosidase
Reference: Murray, R.K., et. al., Harper's Illustrated Biochemistry, 27th ed.,
p. 215
236.
A patient with phenylketonuria has fair skin and eyes due to low levels of:
A. biopterine
C. tryptophan
B. phenylalanine
D. tyrosine
Reference: Murray, R.K., et al., Harper’s Biochemistry, 27th Ed., p. 242.
237.
The preferred amino acid as substrate for gluconeogensis is:
A. alanine
C. asparagine
B. arginine
D. aspartate
Reference: Murray, R.K., et al., Harper’s Biochemistry, 27th Ed., p. 173 .
238.
A patient with mental retardation and pigmentation of connective tissues
has a deficiency of:
A. alanine aminotransferase
C. homogentisate
oxidase
B. aspartate aminotransferase
D. phenylalanine
hydroxylase
Reference: Murray, R.K., et al., Harper’s Biochemistry, 27th Ed., p. 259.
For questions 238-239, refer to the case below.
D.D., a 2 week old male infant has failure to gain weight, nausea and vomiting
after breast feeding. Blood examination revealed increased NH3+.
If D.D. had ↑ blood level of argininosuccinate and friable hair, the deficient
enzyme is:
A. Argininosuccinate synthethase
C. Carbamoyl P
Synthetase 1
B. Argininosuccinate lyase
D. Ornithine
transcarbamoylase
Reference: Murray, R.K., et al., Harper’s Biochemistry, 27th Ed., p. 252.
239.
240.
The immediate treatment for D.D. is:
A. antibiotics administration
C. protein
restriction
B. hippurate administration
D. IV fluids
Reference: Murray, R.K., et al., Harper’s Biochemistry, 27th Ed., p. 251.
29
For questions 241-242, refer to the reaction below.
NH3+
│
R1- C – COO-
O
║
R2- C – COO-
O
║
R1- C – COONH3+
│
R 2- C – COO-
241.
This reaction requires which coenzyme?
A. ATP
C. NAD+
+
B. FAD
D. Vit. B6
Reference: Murray, R.K., et al., Harper’s Biochemistry, 27th Ed., p. 247.
242.
The final products of this reaction are:
A. alanine and pyruvate
B. glutamate and α-ketoglutarate
C. glutamate and oxaloacetate
D. glutamine and oxaloacetate
Reference: Murray, R.K., et al., Harper’s Biochemistry, 27th Ed., p. 248.
243.
The reaction that helps maintain normal acid-base balance is:
A. arginase
C. glutamate dehydrogenase
B. glutaminase
D. glutamine synthetase
Reference: Murray, R.K., et al., Harper’s Biochemistry, 27th Ed., p. 249.
244.
GABA is an inhibitory neurotransmitter because it causes:
A. ↑ Cl- efflux
C. ↑ Cl- influx
B. ↓↓Ca++ efflux
D. ↓ K+ influx
Reference: Murray, R.K., et al., Harper’s Biochemistry, 27th Ed., p. 274.
245.
Which amino acid test dose administration is used to diagnose folate
deficiency?
A. cysteine
C. lysine
B. histidine
D. tryptophan
Reference: Murray, R.K., et al., Harper’s Biochemistry, 27th Ed., p. 254.
246.
Which amino acid test dose administration is used to diagnose pyridoxal
phosphate
deficiency?
A. histidine
C. tryptophan
B. leucine
D. tyrosine
Reference: Murray, R.K., et al., Harper’s Biochemistry, 27th Ed., p. 262.
247.
To prevent obtaining a false-positive test for phenylketonuria, the
screening should be done
how many days after birth?
A. immediately after birth
C. 2 days
B. one day
D. 3 - 4 days
Reference: Murray, R.K., et al., Harper’s Biochemistry, 27th Ed., p. 259.
30
A 10 year old boy with pellagra-like symptoms and a urine finding of ↑
amount of large
neutral amino acids (aminoaciduria). What is the problem with this
patient?
A. homocystinuria
C. glycinuria
B. cystinuria
D. Hartnup
Reference: Murray, R.K., et al., Harper’s Biochemistry, 27th Ed., p. 262.
248.
249.
One of the following statements about the THYROGLOBULIN is TRUE:
A. It is the precursor of calcitonin
B. It is iodinated, glycosylated carbohydrate with a molecular mass of 660
kilodalton
C. It contains 115 tyrosine residues.
D. It is initially synthesized in the apical portion of the cell
Reference: Murray, R.K., et al., Harper’s Biochemistry, 27th Ed., p. 455.
250.
One of the following events is TRUE of the concentration of IODIDE:
A. It is an energy dependent process and is linked to the ATPase
dependent Na-K pump.
B. Thyroperoxidase is essential enzyme for the concentration of Iodide.
C. It involves the iodination of the 3 position of the aromatic ring
D. Phagocytosis and pinocytosis are essential in this process
Reference: Murray, R.K., et al., Harper’s Biochemistry, 27th Ed., p. 456.
251. The half-life of T4 in blood is:
A. 1.5 days
C. 4.5 days
B. 3.5 days
D. 6.5 days
Reference: Murray, R.K., et al., Harper’s Biochemistry, 27th Ed., p. 462.
252.
One of the following statements about THYROID BINDING GLOBULIN is
TRUE:
A. Most of the T4 and T3 is in the unbound form.
B. Thyroxine Binding prealbuminis quantitatively more important.
C. Thyroxine binding globulin binds covalently nearly all thye T3 and T4
D. Estrogens increases the synthesis of thyroid Binding globulin.
Reference: Murray, R.K., et al., Harper’s Biochemistry, 27th Ed., p. 462.
253.
After exposing a jaundiced infant to phototherapy, the following is TRUE of
bilirubin excretion:
A. Bilirubin is converted to bilirubin diglucoronide.
B. Phototherapy provides the sulfates needed to conjugate bilirubin.
C. Bilirubin will be excreted in the feces as mesobilirubin.
D. Increased amounts of urobilin are found in the urine.
Reference: Murray, R.K., et. al., Harper's Illustrated Biochemistry, 26th ed.,
p. 280
Blood Coagulation
254. In normal patients, the fibrinolytic system of the coagulation system is
have the following characteristized by:
A. stabilization of the fibrin molecules by covalent cross-linkage bonding.
B. glycoproteins that regulate coagulation mechanism.
C. components which are thiol-dependent transglutaminases.
D. glycoproteins that contain serine dependent active protease sites.
Reference: Murray, R.K., et. al., Harper's Illustrated Biochemistry, 26th ed.,
p. 604
31
255.
Intracellular calcium is mobilized in the cytoplasm by this PIP2-derived
second messenger.
A. Protein kinase C
C. Inositol diphosphate
B. Inositol triphosphate
D. Phosphatidylinositol
Reference: Murray, R.K., et. al., Harper's Illustrated Biochemistry, 25 th
ed., p. 546
256.
During the follicular phase of the menstrual cycle:
A. there is a sudden increase of estradiol as the follicle enlarges
B. there is proliferation of the endometrial lining
C. the sudden drop in the levels of FSH heralds the end of follicular phase
D. All of the above are TRUE
Reference: Murray, R.K., et. al., Harper's Illustrated Biochemistry, 23rd
Ed., p. 550
257.
Which of the following statement/s is/are TRUE of estrogen?
A. In, pregnancy, more estrone is coming from the placenta
B. Estrogens are formed by the aromatization of androgens in a complex
process that involves 2 hydroxylation steps
C. As much as 50% of estradiol produced during pregnancy are produced
by the ovaries
D. All the statements are TRUE
Reference: Murray, R.K., et. al., Harper's Illustrated Biochemistry, 23rd
ed., p. 450
258.
The following is TRUE about the luteal Phase of the menstrual cycle.
A. There is follicular development
B. The endometrial lining is thickened with the glands.
C. There is low levels of estrogen.
D. FSH and LH levels are high
259.
A.
B.
C.
Which statement/s is/are TRUE about mineralocorticoids?
They are secreted from the zona fasciculata of the adrenal cortex
The final step in the synthesis is conversion of an aldehyde to an alcohol
Synthesis of mineralocorticoids depends on the presence of the
enzyme 18- hydroxylase
D. Mineralocorticoid comes from the parent compound Pregnenolone.
Reference: Murray, R.K., et. al., Harper's Illustrated Biochemistry, 27 th
ed., p. 446.
260.
Which of the following is TRUE regarding the glycemic index?
A. Foods with a low glycemic index cause less fluctuations in insulin
secretion.
B. Foods with a high glycemic index cause rapid fluctuations in insulin
secretion
C. Glucose and galactose have lower glycemic index than non-starch
polysaccharides.
D. Fructose and sugar alcohols are absorbed rapidly and have a higher
glycemic index.
Reference: Murray, R.K., et al., Harper’s Biochemistry, 27th Ed., p. 485.
261.
This water-soluble vitamin requires a specific transport protein:
A. Cobalamin
C. Folic acid
B. Thiamine
D. Riboflavin
Reference: Murray, R.K., et al., Harper’s Biochemistry, 27th Ed., p. 486.
32
262.
Which of the following statements about factors affecting the basal
metabolic rate is TRUE?
A. The basal metabolic rate is higher in females due to greater adiposity
B. Increase in body weight decreases the basal metabolic rate
C. Basal metabolic rate decreases with advancing age
D. Greater muscle tissue means a decrease in basal metabolic rate
Reference: Murray, R.K., et al., Harper’s Biochemistry, 27th Ed., p. 491.
263.
The form of Vitamin A involved in regulation of gene expression is:
A. Retinol
C. Retinyl phosphate
B. Retinaldehyde
D. Retinoic acid
Reference: Murray, R.K., et al., Harper’s Biochemistry, 27th Ed., p. 492.
264.
The main storage form of Vitamin D is:
A. 7-dehydrocholesterol
C. 25-hydroxycholecalciferol
B. Cholecalciferol
D. 1,25 –dihydroxycholecalciferol
Reference: Murray, R.K., et al., Harper’s Biochemistry, 27th Ed., p. 500.
265.
This vitamin participates in the transfer of 1-carbon methyl units:
A. Folic acid
C. Riboflavin
B. Pantothenic acid
D. Thiamine
Reference: Murray, R.K., et al., Harper’s Biochemistry, 27th Ed., p. 500.
266.
The following peptides arise from the proopiomelanocortin gene except:
A. Melanocyte-stimulating hormone
B. Corticotropin-like intermediate lobe peptide
C. Adrenocorticotrophic hormone
D. Gonadotropin releasing hormone
Reference: Murray, R.K., et al., Harper’s Biochemistry, 27th Ed., p. 459.
267.
Which of the following hormones is NOT a glycoprotein?
A. Thyroid stimulating hormone
B. Prolactin
C. Follicle stimulating hormone
D. Human chorionic gonadotropin
Reference: Murray, R.K., et al., Harper’s Biochemistry, 27th Ed., p. 447.
268.
Which of the following is the most utilized pathway of testosterone
synthesis in the human testes?
A. Dehydroepiandrosterone pathway
B. Progesterone pathway
C. Aromatase pathway
D. 21-hydroxylase pathway
Reference: Murray, R.K., et al., Harper’s Biochemistry, 27th Ed., p. 450.
269.
Conversion of cholesterol to pregnenolone in the mitochondria of the
testes is promoted by:
A. Aromatase enzyme
B. Follicle-stimulating hormone
C. Adrenocorticotrophic hormone
D. Luteinizing hormone
Reference: Murray, R.K., et al., Harper’s Biochemistry, 27th Ed., p. 450.
33
270.
Monosaccharides are characterized as:
A. Carbohydrates that can be hydrolyzed to simpler carbohydrates.
B. Are classified based on the number of carbon atoms.
C. Are condensation products of simpler units.
D. Occur in linear configuration.
Reference: Murray, R.K., et al., Harper’s Biochemistry, 27th Ed., Page 112
271.
The glycemic index is a measure of:
A. polarity
C. solubility
B. digestibility
D. amphotericity
Reference: Murray, R.K., et al., Harper’s Biochemistry, 27th Ed., Page 116
272.
The homopolymer structure of starch is called:
A. dextrin
C. glucosan
B. amylopectin
D. amylose
Reference: Murray, R.K., et al., Harper’s Biochemistry, 27th Ed., Page 116
273.
The enzymic hydrolysis of starch by amylase produces:
A. lactose
C. lactulose
B. maltose
D. sucrose
Page 112
274. The phosphorylation of glucose during glycolysis involves:
A. an enzyme located in the mitochondrion
B. GTP as the ATP donor
C. A reversible reaction.
D. an enzyme inhibited allosterically by the product
Page 151
275.
Which of the following is NOT a coenzyme of the pyruvate dehydrogenase
complex?
A. biotin
C. coenzyme A
B. lipoic acid
D. pyridoxine
Page 155
276.
This enzyme of the Krebs Cycle catalyzes a substrate level
phosphorylation reaction:
A. isocitrate dehydrogenase
C. citrate synthase
B. malate dehydrogenase
D. succinate thiokinase
Page 146
Which is not a reaction that synthesizes purine nucleotides?
A. synthesis form amphibolic intermediates
B. phosphoribosylation of nucleobases
C. ribosylation of purine nucleosides
D. phosphorylation of purine nucleosides
Page 301
277.
278.
The multifunctional enzyme involved in the rate limiting reaction during
pyrimidine synthesis includes:
A. IMP dehydrogenase
C. orotidylic decarboxylase
B. dihydroorotase
D. orotate phosphoribosyl transferase
Page 307
34
279.
Gout is a metabolic disorder caused by decreased activity of:
A. hypoxanthine-guanine phosphoribosyltransferase
B. adenosine deaminase
C. glucose 6 – phosphatase
D. xanthine oxidase
Page 308
280.
The endproducts of pyrimidine catabolism are:
A. less water soluble than those of purine catabolism
B. increased in patients with leukemia
C. produced by adenosine deaminase
D. increased with increased dietary intake of nucleotides
Page 308
281.
AcetylCoA derived from glycolysis is:
A. reduced to water in the TCAC
B. converted to NADPH in the hexose monophosphate shunt
C. oxidized to CO2 and H2O in the TCAC
D. converted by pyruvate dehydrogenase to form lactate
Page 134
282.
Which metabolic event occurs during the fed state?
A. There is depletion of metabolic fuels.
B. There is insulin-mediated uptake of glucose in the liver.
C. Extracellular lipoprotein lipase is activated in reponse to insulin.
D. Increased secretion of glucagons by the pancreas.
Page 140
283.
Muscle glycogen is not exported for utilization by peripheral tissues
because of the absence of:
A. gluokinase
C. phosphofructokinse
B. glycogen phosphorylase
D. glucose 6 – phosphatase
Page 142
284. Which metabolic fuel provides the energy during prolonged starvation?
A. glucose
C. amino acids
B. ketone bodies
D. lactate
Page 143
285.
The metabolic pathway that provides the fuel for erythrocytes is:
A. β-oxidation
C. anaerobic glycoslysis
B. Tricarboxylci acid cycle
D. ketogenesis
Page 143
286.
Which statement best describes RNA?
A. The sugar moiety attached to the base is deoxyribose.
B. The guanine content is equal to the cytosine content.
C. RNA contains ribonucloetide uracil.
D. RNA cannot be hydrolyzed by alkali.
Page 314
287.
The adapter molecule in translation is:
A. mRNA
B. tRNA
C. rRNA
D. snRNA
Page 318
35
288.
The human telomere sequence is described as:
A. consisting of short repeat T-G rich sequences
B. looped domains attached to the 3’ end of the chromosome
C. binds sister chromatids during spindle formation
D. involved with the aging when elongated
Page 326
289.
Which of the following features is NOT true of mitochondrial DNA?
A. Encodes the 13 proteins involved in the respiratory chain
B. AGA and AGG are stop codons.
C. Contains 200,000 base pairs.
D. Is circular and double stranded.
Page 331
290.
The synthesis of the forward strand during replication is described as:
A. a continuous process moving from the 3’ to 5’ direction.
B. producing short DNA fragments
C.
D. requiring an RNA primer for each short strand formed
Page 336
291.
Xeroderma pigmentosa results from ultraviolet damage to:
A. the ORIC
C. TG rich palindromes
B. thymine dimers
D. GATC sequence
Page 345
292. The DNA starnd that is transcribed is referred to as the:
A. coding starnd
C. template strand
B. nontemplate strand
D. transcript strand
Page 348
293.
This RNA polymerase II is responsible for the synthesis of:
A. rRNA
C. tRNA
B. mRNA
D. snRNA
Page 351
294.
The rate of transcription may be enhanced by the following sequence
elements, except:
A. TATA box
C. enhancers
B. sigma factor
D. silencers
Page 355
295. In eukaryotes, the sequence that is translated to proteins is called:
A. exons
C. introns
B. cistrons
D. spliceosomes
Page 359
296.
297.
The enzyme that catalyzes the activation of amino acids is:
A. peptidyl transferase
C. aminoacyl-tRNA synthetase
B. RNA polymerase
D. methylguanosine synthase
Page 367
Frameshift mutation results from:
A. deletion of a triplet
C. insertion of a nucleotide
B. insertion of an amino acid
D. deletion of an amino acid
Page 370
36
298.
The role of polyA tail during translation is observed during:
A. elongation
C. Termination
B. initiation
D. modification
Page 372
299.
Which antibiotic prevents binding of aminoacyl-tRNA to the A site?
A. chloramphenicol
C. tetracyclin
B. puromycin
D. rifampicin
Page 378
300.
DNA library that is composed of DNA copies from mRNA populations:
A. chimeric library
C. cDNA
B. Colony
D. genomic library
Page 408
301.
Insulin promotes glucose storage in the liver and muscles by:
A. inhibition of PFK 2.
B. stimulation of G-6-DH.
C. activation of glycogen synthase.
D. increased synthesis of F-2,6 bisP.
Reference: Murray, R.K., et al., Harper’s Biochemistry, 27th Ed., p. 474.
302.
A patient's DNA was analyzed for its chemical characteristics. It was found
to have a high melting temperature which means that the DNA
A. is rich in adenine-thymine base pairs
B. is denatured at a higher temperature
C. will exhibit hypochromicity at this temperature
D. cannot undergo renaturation
Harper's Biochemistry 27th edition, pp. 312-313
Application
303.
A 20 year old male suffering from pneumonia was given a quinolone
antibiotics by his physician. This antibiotics inhibits bacterial DNA
topoisomerase. Which step in replication is inhibited by the quinolone?
A. Unwinding of the double helix.
B. Recognition of the origin of replication
C. Formation of the RNA primer
D. Elongation of the growing DNA polymer
Harper's 27th edition, pp. 339 - 340
Application
304.
A dermatologist advised her patient to avoid too much exposure to
sunlight. Aside from sunburn, ultraviolet light causes what kind of damage
to DNA?
A. Insertion or deletion of nucleotide
B. Oxidative free radical formation
C. Formation of thymine-thymine dimer
D. Cross-linkage between bases
Harper's 27th edition, p. 343
Application
305.
Cells cultured from patients with xeroderma pigmentosum exhibit low
activity to which kind of DNA repair mechanism?
A. Nucleotide excision-repair.
B. Base excision repair.
C. Mismatch repair
D. double-strand break repair
Harper's 27th edition, p. 345
Recall
37
306.
Damage to the mitochondria may result in impaired mitochondrial DNA
synthesis. In mammalian cells, this DNA is synthesized by which type of
DNA polymerase?
A. DNA polymerase alpha
C. DNA polymerase delta
B. DNA polymerase beta
D. DNA polymerase gamma
Harper's 27th edition, p. 336
Recall
307.
Mammalian RNA polymerase II exhibit high sensitivity to the inhibitory
effect of alpha-amanitin, a peptide toxin from a poisonous mushroom.
Which of the following products will be decreased?
A. ribosomal RNA
C. messenger RNA
B. transfer RNA
D. small nuclear RNA
Harper's 27th edition, p. 351
Application
308.
Rifampicin, an anti-TB drug, inhibits DNA-dependent RNA polymerase.
Which of the following steps of gene transcription will NOT be affected by
the drug?
A. Template binding
C. Chain initiation
B. Chain elongation
D. Posttranscriptional modification
Harper's 27th edition, p. 349
Application
309.
A patient is taking steroid drug for his frequent attacks of asthma. Steroids
have anti-inflammatory effects and also regulate gene expression. The
effects of steroids are mediated by DNA elements found in which
transcription control regions of the gene?
A. Coding region
C. Promoter proximal elements
B. Distal regulatory elements
D. CAAT and TATA boxes
Harper's 27th edition, 354 - 356
Application
310.
In beta thalassemia, the beta-globin gene is severely underexpressed
resulting in diminished synthesis of the beta-chain of hemoglobin. This is a
consequence of which posttranscriptional modification of primary RNA
transcript?
A. Faulty exon-intron splicing
B. Addition of poly A tail in the 3' end
C. Incorporation of 7-methylguanosine cap in the 5' end
D. RNA editing in the coding region
Harper's 27th edition, p.362
Application
311.
The genetic code is affected by insertion or deletion of nucleotides in DNA
resulting in frameshift mutation. Which characteristic of the genetic code is
affected by this event?
A. Universal
C. Nonoverlapping
B. Degenerate
D. Unambiguous
Harper's 27th edition, p.366, 370 - 371
Application
312.
Chloramphenicol, a broad spectrum antibiotics, is known to inhibit
peptidyltransferase during translation. What step in protein synthesis is
affected by chloramphenicol?
A. Formation of initiation complex.
B. Elongation of the polypeptide chain.
C. Termination of protein synthesis.
D. Posttranslational modification
Harper's 27th edition, p. 374
Recall
38
313.
The elongation stage of protein synthesis is a multistep process which can
be inhibited by antibiotics. What will happen when the translocation step is
inhibited by streptomycin?
A. Incoming aminoacyl-tRNA cannot bind to the A site
B. Small ribosomal subunit is unable to dissociated from the large
ribosomal subunit.
C. No peptide bond synthesis can take place.
D. Peptidyl-tRNA cannot shift from the A site to the P site.
Harper's 27the edition, p. 375
Application
314.
Some patients receiving methotrexate for treatment of cancer developed
drug resistance. What could be the reason for these?
A. Methotrexate induces amplification of genes for dihydrofolate
reductase.
B. The chemotherapeutic agent inhibits the transport of mRNA from
nucleus to cytoplasm.
C. The drug produces instability to the newly synthesized mRNA.
D. It promoted the use of alternative transcription start site.
Harper's 27th edition, p. 399
Application
315.
Genomic technology oftentimes use probes. What can these molecules
do?
A. They allow visualization of specific DNA fragments.
B. They are used to detect the presence of a specific fragment of
DNA or RNA.
C. They amplify specific DNA sequences.
D. They are used to identify restriction sites.
Harper's 27th edition pp. 409, 420
Recall
316.
Oxygen toxicity can be attributed to the destructive effects of free radicals
generated. These free radicals are removed by which of the following
enzymes?
A. NADPH-cytochrome P450 reductase
C. Superoxide dismutase
B. Peroxidase
D. Catalase
Harper's 27th edition, pp. 97 - 98
Recall
317.
A suicide victim suffered from carbon monoxide poisoning when he was
trapped in the garage with car engine running. Which component of the
respiratory chain is inhibited by the poisonous gas?
A. NADH dehydrogenase of Complex I
B. Succinate dehydrogenase of Complex II
C. Cytochrome c oxidoreductase of Complex III
D. Cytochrome oxidase of Complex IV
Harper's 27th edition, p. 107
Recall
318.
People with bronchial asthma usually has hyperactive airway
bronchoconstriction because of inflammation. Which of the following
eicosanoid is responsible for this?
A. leukotrienes
C. thromboxanes
B. prostaglandins
D. prostacyclins
Harper's 27th edition, p. 122
Application
319.
A premature infant had respiratory distress syndrome after birth. Which of
the following is a major constituent of the lung surfactant?
A. distearoyl cephalin
C. diarachidonyl plasmalogen
B. dioleyl lipositol
D. dipalmitoyl lecithin
Harper's 27th edition, p. 125
Recall
39
320.
Which of the following glycolipids is of considerable biologic interest
because it serves as the receptor in human intestine for cholera toxin?
A. sulfatide
C. glucocerebroside
B. ganglioside (GM3)
D. globoside
Harper's 27th edition, p. 126
Recall
321.
Which of the following steroids although NOT derived from cholesterol
acquire antirachitic property when irradiated with ultraviolet light?
A. cholic acid
C. ergosterol
B. cholecalciferol
D. corticosterone
Harper's 27th edition, p. 127
Recall
322.
Which of the following glycolipids is of considerable biologic interest
because it serves as the receptor in human intestine for cholera toxin?
A. sulfatide
C. glucocerebroside
B. ganglioside (G
323.
How will carnitine deficiency affect the metabolism of glucose?
A. It will lead to hypoglycemia.
B. There is decreased synthesis of NADPH.
C. It will promote glycogenesis
D. There is increased synthesis of glucuronic acid
Harper's 27th ed., p 187 - Application
324.
A patient suffering from hypoalbuminemia will affect the transport of which
lipid in the plasma?
A. cholesterol
C. unesterified fatty acid
B. triacylglycerol
D. phospholipid
Harper's 27th ed., 187 - Application
325.
Lack of carnitine in the mitochondria will directly and immediately affect
which aspect of fatty acid oxidation?
A. activation of fatty acid
B. transport of fatty acid into the mitochondrial matrix
C. oxidation of fatty acyl CoA
D. formation of acetyl CoA
Harper's 27th ed., 187 - Application
326.
Infants born without peroxisomes will suffer from neurologic symptoms
because of the accumulation of what type of fatty acid?
A. short chain saturated fatty acid
B. medium chain saturated fatty acid
C. long chain saturated and unsaturated fatty acid
D. very long chain unsaturated fatty acid
Harper's 27th ed, p 194 - Recall
327.
The basic form of ketosis occurs in starvation which involves depletion of
carbohydrates coupled with increased:
A. mobilization of free fatty acid
B. fatty acid synthesis
C. triacylglycerol synthesis
D. cholesterol metabolism
Harper's 27th ed., 194 - Application
40
328.
A patient taking aspirin because of chronic arthritis will have decreased
synthesis of which of the following?
A. leukotrienes
C. prostaglandin
B. ketone bodies
D. glycosphingolipid
Harper's 27th ed., 196 - Application
329.
Deficiency of biotin will result in decreased formation of which of the
following?
A. acyl carrier protein
C. phosphatidic acid
B. malonyl CoA
D. ceramide
Harper's 27th ed., 197 - Application
330.
Essential fatty acid deficiency may be exacerbated by increased intake of
which of the following foodstuffs?
A. margarine
C. coconut oil
B. butter
D. vegetable oil
Harper's 27th ed., 203 - 204 - Application
331.
Which of the following enzymes that degrade phosphoglycerides is one of
the major toxins secreted by pathogenic bacteria?
A. Phospholipase A1
C. Phospholipase A2
B. Phospholipase C
D. Phospholipase D
Harper's 27th ed., p. 212 - Recall
332.
Enlarged liver and spleen, erosion of long bones, mental retardation in
infants are seen in which of the following lysosomal storage diseases?
A. Tay Sach's disease
C. Gaucher's disease
B. Fabry's disease
D. Farber's disease
Harper's 27th ed., 215 - Application
333.
Patients with lipoprotein lipase deficiency will affect lipoprotein metabolism
in what way?
A. Increased formation of lipoprotein remnants
B. Increased degradation of VLDL
C. Decreased synthesis of HDL
D. Decreased triglyceride hydrolysis in chylomicron
Harper's 27th ed., 219 - 220 - Application
334.
Administration of simvastatin will decrease the activity of which of the
following enzymes involved in cholesterol synthesis?
A. HMG-CoA synthase
C. Squalene oxidocyclase
B. HMG-CoA reductase
D. Mevalonate kinase
Harper's 27th ed., p 231 - Recall
335.
During protein analysis a new amino acid was found which has a pK3
value, 3 units lower than that of cysteine. This so-called 21st amino acid
is:
A. aluminocysteine
C. manganocysteine
B. selenocysteine
D. mangesimosysteine
Harper's 27th ed., p. 14 - Recall
336.
A protein isolated from a cancerous tissue absorbs visible light in the
region of 280 nm. Which amino acid component of the protein is
responsible for this?
A. glycine
C. valine
B. arginine
D. tryptophan
Harper's 27th ed., p. 18 - Recall
41
337.
Hemoglobin has the ability to neutralize protons released by carbonic acid.
Which of the following amino acids is responsible for this?
A. lysine
C. histidine
B. glutamine
D. isoleucine
Harper's 27th ed., p. 45 - Application
338.
Proteins must be purified prior to analysis. Size exclusion chromatography
separates proteins based on their Stokes radius which is a function of:
A. molecular mass and shape
B. solubility characteristics
C. chemical interactions with other groups
D. denaturation and renaturation potential
Harper's 27th ed., p. 21 - 22 -- Application
339.
Scurvy which is due to vitamin C deficiency results in the formation of
defective collagen characterized by:
A. decreased number of hydroxyproline and hydroxylysine
B. increased in the number of covalent cross-links
C. decreased synthesis of disulfide bridges
D. increased interactions with copper and other minerals
Harper's 27th ed., p 39 - Application
340.
Tissue aminotransferases reversibly interconvert which of the following
three amino acids and their corresponding alpha-keto acids?
A. serine, threonine and tyrosine
B. lysine, arginine and histidine
C. glycine, alanine and isoleucine
D. valine, leucine and isoleucine
Harper's 27th ed., p. 243 - Application
341.
Which of the following is NOT a part of the stages occuring in urea
biosynthesis:
A. transamination
B. transaminidation
C. oxidative deamination of glutamate
D. ammonia transport
Harper's 27th ed., 247 - Application
342.
Deficiency of which of the following coenzymes will result in decreased
transamination of amino acids?
A. thiamine pyrophosphate
C. pyridoxal phosphate
B. biotin
D. tetrahydrofolate
Harper's 27th ed., 248 - Application
343.
Transdeamination refers to the concerted reactions catalyzed by which of
the following pair of enzymes?
A. glutamate aminotransferase and glutamate dehydrogenase
B. glutamate dehydrogenase and glutamine synthetase
C. L-amino acid oxidase and cyseine desulfurase
D. Serine dehydrase and glutamate transaminase
Harper's 27th ed., 248 - Application
42
344.
Of the six participating amino acids in urea synthesis which of the
following functions solely as an enzyme activator?
A. ornithine
C. citrulline
B. argininosuccinate
D. N-acetylglutamate
Harper's 27th ed., p. 249 - Recall
345.
Ammonia intoxication impairs neuronal functions of the brain because it:
A. impairs the synthesis of glutamine
B. inhibits the activation of asparagine synthetase
C. depletes the levels of alpha-ketoglutarate of the Kreb's cycle
D. produces respiratory alkalosis
Harper's 27th ed., p. 248 - Application
346.
Which of the following chemical reactions typically initiates amino acid
catabolism?
A. glutamate dehydrogenation
C. transmethylation
B. transamination
D. transulfuration
Harper's 27th ed., p. 254 - Recall
347.
Alkaptonuria, an inborn error of tyrosine metabolism, is characterized by
darkening of the urine on exposure to air. This is due to oxidation of
excreted:
A. homogentisic acid
C. maleylacetoacetate
B. hydroxyphenylpyruvate
D. fumarate
Harper's 27th ed., p. 259 - Application
348.
Maple syrup urine disease is a metabolic disorder of branched-chain
amino acid catabolism. This is due to which of the following?
A. lack of leucine, isoleucine and valine aminotransferase complex
B. defective alpha-keto acid decarboxylase
C. mutation of dihydrofolate reductase
D. inability to attached the biotin coenzyme
Harper's 27the ed., p. 265 - Application
349.
S-adenosylmethionine is important in the body because it is involved in
which of the following reactions?
A. transulfuration
C. methylation
B. acetylation
D. conjugation reaction
Harper's 27th ed., p 274 - Recall
350.
Which is a mechanism of oncogene activation? page 794 , MPL = 1
A. chromosome translocation C. promotion
B. initiation
D. proliferation
351.
TRUE of oncogene activation of chronic myelogenous leukemia: page
794, MPL= .25
A. It involves gene re-arrangement of chromosome 9 and 14
B. The mutation results to fusion of BCR-ABL gene
C. It is a recessive type of mutation
D.It results to overexpression of c-myc.
352.
Stage of chemical carcinogenesis which is genotoxic and irreversible:
page 790, MPL= 1
A. initiation
C. progression
B. promotion
D. mutation
43
353.
Philadelphia chromosome in chronic myelogenous leukemia is due to:
page 794, MPL= .5
A. gene rearrangement of chromosome 9 and 22
B. fusion of myc-ras
C. activation of c-myc
D. translocation of chromosome 8 and 14
354.
Which of the following molecules function to transfer information from the
nucleus to
the cytoplasm? (page 309, MPL=1)
A. DNA
C. tRNA
B. mRNA
D. proteins
355.
Three types of RNA involved in comprising the structural and functional
core for protein synthesis, serving as a template for translation, and
transporting amino acid, respectively, are:
(Page 341, MPL=0.5)
A. mRNA, tRNA, rRNA
C. rRNA, tRNA, mRNA
B. tRNA, mRNA, rRNA
D. rRNA, mRNA ,tRNA
356.
What is added to the 3'-end of many eukaryotic mRNAs after
transcription?
(Page 356, MPL=0.75)
A. introns
B. a poly A tail A
C. a cap structure, consisting of a modified G nucleotide
D. the trinucleotide 5'-CCA
The regions of DNA in a eukaryotic gene that encode a polypeptide
product are called:
(Page 352, MPL=1)
A. hnRNAs
C. enhancers
B. exons
D. leader sequences
357.
358.
A molecule containing sequences from two different species is known as a
(Page 419, MPL = 1)
A. clone
B. cosmid
C. plasmid
D. chimeric molecule
359.
The sequence of a gene that is transcribed but excised before translation:
(Page 365, MPL = 1)
A. enhancer
B. exon
C. intron
D. repressor
360.
Which of the following statements is FALSE regarding a genomic library?
(Page 408, 409) MPL = 0.5
A. cDNA libraries are represented only by exons
B. genomic DNA libraries are represented only by introns
C. it is a collection of cloned fragments that represents the entire genome
D. libraries are prepared by performing partial digestion of total DNA with
a restriction enzyme
44
361.
Which of the following statements is FALSE regarding the genetic code?
(Page 366, MPL = 0.5)
A. Some amino acids are encoded by several codons
B. In general, the third nucleotide in a codon is less important than the first
two in coding for a specific amino acid
C. It is universal with the exception of tRNA molecules in mitochondria
D. The reading of the genetic code may involve overlap of codons in
some instances
362.
The genetic code is said to have this property because multiple codons
code for the same amino acid. (Page 366, MPL = 1)
A. degenerate
B. unambiguous
C. universal
D. no punctuation
363.
POMC gives rise to all of the following substances, except:
[p. 88] MPL- 1.0
A. ACTH
B. endorphins
C. α-MSH
D. renin
364.
Which of the following structural motifs is important for the transcriptional
activation produced by some steroid-bound hormones? [pp. 393-396]
MPL- 1.0
A. zinc fingers
B. gly-x-y
C. immunoglobulin folds
D. EF hands
365.
Irreversible activator of adenylyl cyclase causing ribosylation of the
αstimulatory - subunit of the G-protein: [p. 567] MPL- 0.5
A. pertussis toxin
B. phosphodiesterase
C. cholera toxin
D. calcium
366.
Catecholamines undergo oxidative deamination by the action of the
following
enzyme: [25th ed., pp. 590-591] MPL – 0.5
A. tryptophan-5hydroxylase
C. COMT
B. MAO
D. tyrosine hydroxylase
367.
Estrogenic hormone which is predominant during pregnancy.
[p. 450] MPL – 1.0
A. estradiol B. estriol
C. estrone
D. diethylstilbestrol
368.
Leprechaunism is a rare condition characterized by elfin facies, low birth
weight, decreased muscle mass and subcutaneous fat due to which of the
following defects?
[25th ed., pp. 622-623] MPL- 0.5
A. growth hormone deficiency
B. thyroid hormone receptor defect
C. growth hormone receptor defect
D. insulin receptor defect
45
369.
Growth hormone is synthesized and stored in large amounts in the
anterior pituitary gland. Which of the following statements accurately
describes GH?
[25th e4d., pp. 551-553] MPL- 0.5
A. it is secreted continuously
B. it stimulates cartilage and bone growth via somatomedin C
C. it has a proinsulin-like effect in addition to its other actions
D. synthesis is stimulated by the action of somatostatin
370.
In congenital adrenal hyperplasia involving 21-hydroxylase deficiency,
which of the following is true? [25th ed., p.587] MPL- 0.5
A. cortisol accumulates
B. 17-OH-progesterone is an effective treatment
C. most patients suffer "salt-wasting" due to lack of aldosterone
D. levels of ACTH in blood decreased
371.
Which of the following is a direct effect of parathyroid hormone in
regulating calcium metabolism? [25th ed, p. 570] MPL – 0.5
A. increases bone resorption
B. increases intestinal absorption of calcium
C. decreases renal excretion of calcium
D. decreases bone mineralization
372.
The bond that links together complementary base pairs along DNA
strands :
A. hydrogen
B. glycosidic
C. phosphodiester
D. peptide
(Page 303, MPL = .5)
373.
Poly A tail and 7 methyl guanosine cap are features of:
A. DNA
B. mRNA
C. tRNA
D. rRNA
(Page 309, MPL = .5 )
374.
DNA synthesis occurs during this phase in the cell cycle :
A. G1
C. G0
B. G2
D. S phase
(Page 333, MPL = .5)
375.
This drug blocks the reduction of dihydrofolate to tetrahydrofolate:
A. allopurinol
C. methotrexate
B. colchicine
D. sulfa drugs
(Page 296, MPL= .5)
376.
A blot transfer procedure for RNA analysis. (Page 409, MPL = 1)
A. Southern blot
C. Northern blot
B. Western blot
D. Southwestern blot
377.
A manual enzymatic method of DNA sequencing. (Page 410, MPL = 0.5)
A. Sanger’s method
B. Restriction fragment length polymorphism
C. Maxam-Gilbert method
D. Polymerase chain reaction
46
378.
This is a method of amplifying a target sequence of DNA. (Page 410, MPL
=1
A. Flourescence in situ hybridization C. Sanger’s method
B. Polymerase chain reaction
D. Microarray technology
379.
A two year old boy, SHORT for his age, presented with leg edema and
abdominal swelling. He was weaned from breast milk at one year of age,
because of the coming of another baby. He was fed rice or lugaw
everyday mixed with condensed milk. The edema is caused by:
A.
Lack of protein in his diet
B.
Excessive intake of water
C.
Kidney failure
D.
Presence of intestinal parasites
Recall
Reference: Harper's Illustrated Biochemistry 27th ed., Murray, Granner
& Rodwell 2006 The macgraw-Hill Companies, Inc., p. 241, par. 1, p.
487, par. 3
380.
Catabolism of amino acids requires the participation of transaminase
enzymes. What
Coenzyme form of vitamin B is an important participant in this reaction?
A.
Thiamin pyrophosphate
B.
Nicotinamide adenine dinucleotide
C.
Tetrahydrofolic acid
D.
Pyridoxal phosphate
Recall
Reference: Harper's Illustrated Biochemistry 27th ed., p. 247 par. 2 , p.
498 - 499
381.
In marasmus, the affected person has very little fat reserves and there is
wasting of
Muscles. The person is said to be in :
A.
Positive nitrogen balance
B.
Negative nitrogen balance
C.
Nitrogen balance
D.
Equilibrium
Recall
Reference: Harper's Illustrated Biochemistry 27th ed., p. 487 par. 3
382.
Persons who consume large amounts of unleavened whole-wheat
products may
Experience calcium deficiency because:
A. Wheat is poor in calcium
B. Wheat contains phytic acid which prevents calcium absorption
C. The wheat is deficient in vitamin D
D. Absorbable form of calcium is not generated
Application
Reference: Harper's Illustrated Biochemistry 27th ed., p. 485 par. 5
47
383.
An elderly woman is believed to be suffering from vitamin B12 deficiency.
Which of the
Following can be the basis for this statement?
A. She has been diagnosed to be deficient in cobalt
B. She is a vegan
C. She has methylmalonic aciduria
D. Any of the above
Application
Reference: Harper's Illustrated Biochemistry 27th ed., p. 499-500
384.
The patient was in coma. Since diabetes was ruled out, how does one
explain the condition?
A. Patient has ammonia intoxication because of the liver pathology
B. Coma is due to marked ketosis
C. Acid-base balance of the patient is disturbed
D. There is increase urea production
Application
Reference: Harper's Illustrated Biochemistry 27th ed., p. 248 par. 4
385.
The obese matron was instructed by her physician to alter her diet and eat
only protein foods to bring down her weight. The matron was afraid
however that the absence of carbohydrates in her meals will cause
hypoglycemia and give her more problems. The physician was certain
she does not have to worry about this because:
A. She's going to cheat on her diet anyway
B. The excess body fat will be converted to glucose
C. Amino acids in excess are converted to glucose
D. She has a lot of glycogen slores
Application
Reference: Harper's Illustrated Biochemistry 27th ed., p. 172 par. 2, p.
175 summary, P. 269 summary
386.
Deficiency of iron results in which of the following?
A.
Failure to metabolize proteins
B.
Heme-containing enzymes remain inactive
C.
There is liver dysfunction
D.
Individual goes into ATP production using fats
Recall
Reference:
387.
Harper's Illustrated Biochemistry 27th ed., p. 504 Table
An infant with genetic deficiency of glucose-6-phosphatase also known as
von Gierke's disease will exhibit this clinical sign:
A.
Poor exercise tolerance
B.
Muscle dystrophy
C.
Hypoglycemia & lactic acidemia
D.
Glycogen with few branch points
Recall
Reference:
Harper's Illustrated Biochemistry 27th ed., p. 166 Table
48
388.
In the absence of creatine kinase enzymes, an athlete would have
problem in:
A.
Oxidizing creatine
B.
Releasing ATP from biological oxidation
C.
Aerobic glucose oxidation
D.
Generating ATP reserves for muscle contraction
Recall
Reference:
389.
Harper's Illustrated Biochemistry 27th ed., p. 583 par. 1-3
A young woman already in her fifth month of a carbohydrate and lipid-free
diet was advised by her physician to increase the amount of fats in her
diet because she appears to be suffering from vitamin A deficiency despite
her regular vitamin intake. Which of the following complaints presented by
the woman supports the diagnosis?
A.
She has lesions of the corner of the mouth
B.
Her wounds take longer time to heal
C.
She finds it hard to see in the dark when in a movie theater
D.
She has this sensation of "burning feet".
Application
Reference: Harper's Illustrated Biochemistry, 27th edition, p. 491, par.
4, line 3
390.
Tests done on a two month-old baby showed deficiency in enzymes of the
Uronic Acid Pathway. What clinical manifestation may be observed?
A.
Elevated urinary glucuronic acid
B.
Yellow coloration of the skin
C.
High direct-reacting bilirubin in plasma
D.
Accumulation of heme
Application
Reference: Harper's Illustrated Biochemistry, 27th edition, p. 289 par.
2, p. 290 par. 2
391.
A 35 year old Muntinlupa inmate was confined to 6 weeks of isolation
because of bad behavior. Although he was given regular meals, he
stayed in the underground cell for the duration of the 6 weeks. If the
prisoner became deficient in vitamin D because of this, what signs should
the attending physician look for? In the inmate:
A.
There may be kidney stones formation
B.
The inmate may complain of tiredness
C.
He can easily be fractured
D.
Serum calcium will be extremely low
Application
Reference: Harper's Illustrated Biochemistry, 27th edition, p. 492 - 493
392.
Thiamine deficiency can result in Wernicke- Korsakoff's syndrome. The
absence of the vitamin prevents the conversion of pyruvate to acetyl coa
on a high carbohydrate diet. Which compound would be found to be high
in concentration in the plasma?
A.
Glucose
B.
Lactate & pyruvate
C.
Acetyl coa
D.
Alcohol
49
Application
Reference: Harper's Illustrated Biochemistry, 27th edition, p. 496- 497
393.
Important in the synthesis of collagen, this vitamin serves as cofactor of
the mixed function oxygenase enzymes, proline & lysine hydroxylases:
A.
Vitamin B1
B.
Vitamin B2
C.
Vitamin C
D.
Folic acid
Recall
Reference:
9
394.
Unlike other vitamins, niacin is a semi-essential vitamin because it can be
synthesized from this amino acid:
A.
Proline
B.
Phenylalanine
C.
Tryptophan
D.
Histidine
Recall
Reference:
395.
Harper's Illustrated Biochemistry 27th ed., p. 185 last par.
A patient was tested for transketolase activity of the erythrocyte lysate.
Results showed markedly low activity of the enzyme, consistent with the
physicians impression of deficiency of this vitamin:
A.
Vitamin B2
B.
Vitamin B3
C.
Vitamin B5
D.
Vitamin B1
Application
Reference:
397.
Harper's Illustrated Biochemistry 27th ed., p. 498, par. 1
Activity of the aldose reductase enzyme in the lens of the eye increases in
the presence of high concentration of glucose. This initiates the
conversion of glucose to a compound that has been linked to osmotic
damage and cataract formation. What compound is this?
A.
Mannitol
B.
Galactitol
C.
Ribitol
D.
Sorbitol
Recall
Reference:
396.
Harper's Illustrated Biochemistry 27th ed., p. 242 par. 3, line
Harper's Illustrated Biochemistry 27th ed., p. 497, par. 1
Inorganic iron is best absorbed by the body in the ferrous form. What is
the rationale behind the addition of vitamin C in many iron preparations?
A.
Vitamin C can take the place of iron
B.
Absorption of other minerals is prevented
C.
Vitamin C chelates iron
D.
Vitamin C keeps the iron in reduced (ferrous) state
Application
Reference: Harper's Illustrated Biochemistry 27th ed., p. 485 - 486
50
398.
In a routine medical check up, an executive showed elevated plasma
levels of lactic acid dehydrogenase enzymes 1 & 2 and low levels of LDH
5. This prompted his physician to refer him to a cardiologist because:
A. Low levels of LDH 5 indicated a clogged vessel
B. It is part of the medical check up
C. Increase levels of LDH 1 & 2 is indicative of myocardial infarction
D. Angioplasty was needed
Application
Reference: Harper's Illustrated Biochemistry 27th ed., p. 57
399.
A tourist from the Caribbean fell ill and was diagnosed to have malaria.
He was given primaquine upon admission to the hospital and was thought
to be recuperating when he showed signs of hemolytic anemia. Tests
showed he had glucose 6- phosphatase deficiency. What is the
biochemical explanation for the hemolysis?
A. The malarial parasite had invaded the rbc
B. Inactivity of the Hexose Monophosphate Shunt prevented the
NADPH - reduction of membrane glutathione
C. The drug oxidized membrane proteins
D. Absence of G6PD initiated release of peroxides
Application
Reference: Harper's Illustrated Biochemistry 27th ed., p. 177 par. 2, p.
183 par. 2
A. Metabolic screening of a newborn showed absence of galactose
uridyltransferase enzyme. What should be the dietary
recommendation?
A.
Galactose should not be included in the diet
B.
Galactose - free and lactose -free milk should be given
C.
Newborn should be breastfed
D.
Liquid protein preparation instead of milk should be given
Recall
Reference: Harper's Illustrated Biochemistry 27th ed., p. 186 par. 2
400.
A born-again vegetarian was diagnosed to be suffering from vitamin B12
deficiency. Which
Of the following suggestions should be followed?
A. The vegetarian should include meat in the diet
B. He should be tested for pernicious anemia
C. The born-again can take in vitamin B12 preparation to meet the
requirement
D. Intrinsic factor should be released from bonding
Recall
Reference: Harper's Illustrated Biochemistry 27th ed., p. 499 - 500
401.
A woman had just given birth by ceasarean section. For fast recuperation
she should be
Advised to:
A.
Eat nutritious and protein-rich foods
B.
Rest in bed until the incision heals
C.
Include more carbohydrates in the diet
D.
Drink a lot of water
51
Recall
Reference: Harper's Illustrated Biochemistry 27th ed., p. 487 - 488
Questions # 402-404
A 32 year old married woman consulted at the OPD because of increasing thirst
and frequent urination. She is 5 feet 2 inches tall and weighs 168 lbs.
402.
Which of the following statements best described the metabolic status in
this patient?
a. Her energy expenditure is greater than food intake hence she is
diuresing
b. She is obese and she has increased mortality
c. The (BMR) basal metabolic rate of this patient is higher compared to a
man of the same body weight
d. She is frequently urinating because she has increased protein reserves
Application
Reference: Harper's Illustrated Biochemistry 27th ed., p. 486
403.
The BMI of the patient is:
A.
25
B.
27
C.
29
D.
31
Application
Reference: Harper's Illustrated Biochemistry 27th ed., p. 486 par. 2
404.
You want to give this patient a 1,500 cal/day diet because you want her to
lose weight. How
Do you formulate an appropriate plan for this patient?
A.
Carbohydrate - 55% Fats = 35%
Protein = 10%
B.
Carbohydrate - 30% Fats = 20%
Protein = 50%
C.
Carbohydrate - 0% Fats = 40%
Protein = 60%
D.
Carbohydrate - 40% Fats = 40%
Protein = 20%
Application
Reference: Harper's Illustrated Biochemistry 27th ed., p. 132
405.
The edema of a patient with kwashiorkor should respond best when
prescribed with:
A.
Diet rich in protein
B.
Diuretics
C.
Bed rest
D.
Virgin coconut oil
Application
Reference: Harper's Illustrated Biochemistry 27th ed., p. 487
52
406.
A 32 year old physician suffered a 3rd degree burn. He stayed in the
hospital for almost one month. What can be said of his metabolic
condition
A. The excretion of nitrogenous compounds is less than the dietary intake
B. Negative nitrogen balance
C. Positive nitrogen balance
D. There is net retention of the nitrogen in the body as protein
Application
Reference: Harper's Illustrated Biochemistry 27th ed., p. 487
407.
The human body, even at rest, uses up energy. This amount of energy
expenditure is termed as:
A.
Respiratory Quotient
B.
Basal Metabolic Rate
C.
Respiratory Burst
D.
Body Mass Index
Recall
Reference:
408.
Harper's Illustrated Biochemistry 27th ed.,
p. 486
Uptake of glucose in these tissues is controlled by the hormone insulin:
A.
Liver & kidneys
B.
Adipose tissue & muscle
C.
Muscle & intestines
D.
Liver & rbc
Recall
Reference: Harper's Illustrated Biochemistry 27th ed., p. 140, par. 3,
line 1
409.
Between meals, the blood glucose concentration of the body is
maintained through
These metabolic pathways:
A.
Glycolysis & gluconeogenesis
B.
Glycogenesis & glycogenolysis
C.
Glycolysis & glycogenolysis
D.
Glycogenolysis & gluconeogenesis
Recall
Reference: Harper's Illustrated Biochemistry 27th ed., p. 134, par. 4,
line 8
410.
During continuous anaerobic physical activity, e.g. When working out in a
gym, this metabolic pathway is used by the muscles
A.
Gluconeogenesis
B.
Glycolysis
C.
Hexose Monophosphate Shunt
D.
Glycogenesis
Recall
Ref. Harper's Illustrated Biochemistry 27th ed., p. 151, p. 158
53
411.
When tested, a young woman, complaining of fatigability and poor
exercise tolerance was found to have only one sixth the amount of
carnitine found in others who had no muscle disease. This means that the
above symptoms she is experiencing is due to:
A.
Inability of the body to synthesize lipids
B.
Inhibition of ATP production
C.
Failure of fatty acids to enter - oxidation
D.
Impairment of the mitochondria
Recall
Ref. Harper's Illustrated Biochemistry 27th ed., p. 187 par. 4
412.
Newborn infants can generate more heat needed by the body because
they contain
Thermogenin, an uncoupling protein. This protein exerts its effect by:
A. Increasing the amount of substrate oxidized
B. Preventing heat loss
C. Dissociating respiratory chain oxidation from ATP synthesis
(phosphorylation)
D. Activating respiratory chain enzymes
Application
Reference: Harper's Illustrated Biochemistry 27th ed., p. 107, par. 4, p.
228 - 229
413.
Erythrocytes are hemolyzed when there is impairment of the Pentose
Phosphate Pathway and this is manifested in a person as hemolytic
anemia. This defect is due to:
A.
Lack of NADPH from the Pentose Phosphate Pathway
B.
Non-synthesis of ATP
C.
Failure to release ribose
D.
Defect in the membrane transport
Application
Reference: Harper's Illustrated Biochemistry 27th ed., p. 183- 184, par.1
414.
A 30 year old patient was diagnosed to be suffering from a mitochondrial
disease that specifically involves defect in mitochondrial energy
transduction.
Which of the following signs/symptoms will be evident?
A.
High body mass index (BMI), hyperactivity, increased appetite
B.
Hyperventilation, clammy sweat
C.
Muscle weakness, excessive perspiration, and elevated BMR
(Basal metabolic
Rate)
D.
Lack of appetite, hypoventilation
Application
Reference:
415.
Harper's Illustrated Biochemistry 27th ed., p. 100, par. 2
A newborn infant tested positive in the phenylketonuria screening. The
attending pediatrician ordered a change in the infant milk to one which
contains negligible amount of:
A.
Lactose
B.
Proteins
C.
Fats
D.
Glucose
54
Application
Reference:
416.
Harper's Illustrated Biochemistry 27th ed., p. 259, par. 3
A drug used as an adjunct in the treatment of obesity inhibits pancreatic
lipase. Following its intake, which of the following is expected to be
excreted in great amount in the feces?
A.
Cholesterol
B.
Triacylglycerols
C.
Free fatty acids
D.
Bile salts
Application
Reference: Harper's Illustrated Biochemistry 27th ed., p. 483, par. 4, p.
484 diagram
417.
In the mucopolysaccharidoses, glycosaminoglycans are not degraded
due to the absence of lysosomal hydrolases. Individuals suffering from
this can be identified because of:
A. Excessive accumulation and urinary excretion of the
proteoglycans
B. Presence of enzyme fragments in the urine
C. Release of the core proteins in urine
D. Increase turnover of affected cells
Recall
Reference:
418.
Harper's Illustrated Biochemistry 27th ed., p. 555 par. 5
In the rare disease, analbuminemia, there is complete lack of serum
albumin. The affected
Individual presents with this:
A.
Extreme edema
B.
Edema & presence of free fatty acids
C.
Accumulation of triglycerides
D.
Elevated plasma cholesterol
Recall
Reference: Harper's Illustrated Biochemistry 27th ed., p. 592 par. 1, p.
187 par. 3 P. 711 Clinical Correlation , Textbook of Biochemistry, 5th ed.,
Thomas Devlin, Editors, 2002 Wiley-Liss
419.
Metabolic screening was requested for a week-old newborn because the
grandmother had noticed the dark stains on the wet diaper whenever she
attended the infant. Tests identified the condition as alkaptonuria. What
urinary metabolic should the physician request in the laboratory tests?
A.
Tryptophan
B.
Tyrosine
C.
Homogentisic acid
D.
Melanin
Recall
Reference:
Harper's Illustrated Biochemistry 27th ed., p.259 par. 2
Textbook of Biochemistry, 5th ed., Thomas Devlin, p. 801,
Clinical Correlation
55
420.
Skeletal muscles do not have the enzyme glucose-6-phosphatase. This
means:
A.
They cannot degrade glucose
B.
Glucose cannot be synthesized
C.
ATP cannot be generated
D.
Glycogen is stored in big amount
Recall
Reference:
421.
L-xylulose reductase catalyzes the reduction of xylulose to xylitol. In
affected individuals,
Absence of this enzyme results in:
A.
Formation of cataracts
B.
Failure to synthesize nucleic acids
C.
Excretion of large amounts of urinary pentose
D.
Presence of skeletal deformities
Recall
Reference:
422.
Harper's Illustrated Biochemistry 27th ed., p. 161, p. 167
Harper's Illustrated Biochemistry 27th ed., p. 184 par. 2
When there is restricted intake of carbohydrates as in starvation and
uncontrolled diabetes mellitus, ketosis results because of:
A.
Change in ph of the body
B.
Failure to oxidize glucose
C.
Mobilization of fatty acids
D.
Inactivity of buffer systems
Application
Reference: Harper's Illustrated Biochemistry, 27th ed., p. 194 last par. ,
p.195 , 2nd par.
(summary)
423.
Measurement of hba1c or glycosylated hemoglobin provides valuable
information for the management of diabetes mellitus because it:
A. Is elevated in diabetes mellitus
B. Reflects the mean glucose concentration over the preceding 6 - 8
weeks
C. Is formed by reaction with hba
D. Is released when rbc dies
Application
Reference: Harper's Illustrated Biochemistry 27th ed., p. 48, par.1
424.
Inherited enzyme deficiencies of the Krebs-Henseleit cycle always present
with this observation:
A.
Increase urea production
B.
Formation of organic metabolites
C.
Ammonia intoxication
D.
Depletion of citrulline
Application
Referenece: Harper's Illustrated Biochemistry 27th ed., p. 251, par. 4
56
425.
The urine-soaked diaper of a week old baby was noted by the mother to
smell like burnt sugar. Her concern turned into worry when the
pediatrician told her the baby may be suffering from maple syrup urine
disease. To confirm this, activity of the keto acid decarboxylase complex
has to be measured. Which amino acids will have increased
concentration in plasma and urine?
A.
Serine, threonine & hydroxyproline
B.
Cysteine, cystine, methionine
C.
Valine, leucine & isoleucine
D.
Glycine, alanine & asparagine
Application
Reference:
426.
Harper's Illustrated Biochemistry 27th ed., p. 265, par. 2
Calcium deficiency manifestations may also be seen in people who
consume:
A.
Large amounts of unleavened whole wheat products
B.
High concentrations of fatty acids
C.
High amounts of oxalates
D.
All of the above
Application
Referenece: Harper's Illustrated Biochemistry 27th ed., p. 265, par. 2
427.
Milk intolerance, common among Asian populations and characterized by
an osmotic type of diarrhrea following the intake of milk is attributed to
deficiency of:
A.
Pancreatic amylase
B.
Lactase
C.
Sucrase
D.
Maltase
Recall
Ref. Harper's Illustrated Biochemistry 27th ed.,
428.
Rate-limiting reaction in cholesterol synthesis which is inhibited by the
"statins" group of
Drugs is catalyzed by this enzyme:
A.
Cholesterol esterase
B.
Squalene cyclase
C.
Mevalonate kinase
D.
Beta hydroxyl-beta methyl glutaryl coa reductase
Recall
Ref. Harper's Illustrated Biochemistry 27th ed.,
429.
p. 483, par. 1, line 3
p. 231, par. 2
All the carbon atoms of cholesterol are synthesized from this common
metabolite:
A.
Alanine
B.
Acetyl coa
C.
Glucose
D.
Palmitate
Recall
Ref. Harper's Illustrated Biochemistry 27th ed.,
p. 230, par. 3
57
430.
Carbohydrates, lipids & proteins can be oxidized to acetyl coa or specific
metabolites intermediates using a final common pathway. The pathway
functions in gluconeogenesis, lipogenesis and interconversion of amino
acids. This pathway is known as:
A.
Glycolysis
B.
Hexose Monophosphate Shunt
C.
Citric Acid Cycle
D.
Glucuronic Acid Pathway
Recall
Ref. Harper's Illustrated Biochemistry 27th ed., p. 145, par. 1
431.
In the capillaries, hemoglobin releases oxygen and accepts carbon
dioxide. The process generates protons e which can then lower the blood
ph. The buffering action of hemoglobin will prevent this from taking place.
What amino acid residues in hemoglobin are most important in this shift in
ph buffering effect at normal blood ph?
A.
Arginine
B.
Lysine
C.
Histidine
D.
Glutamate
Recall
Ref. Harper's Illustrated Biochemistry 27th ed., p. 43
432.
The highest number of energy released as ATP is obtained by complete
oxidation of this compound to carbon dioxide and water
A.
An amino acid
B.
A triglyceride
C.
Glucose
D.
A nucleic acid
Application
Ref. Harper's Illustrated Biochemistry 27th ed.,
p.189, par.1
433.
p. 142, table 16-1,
Laboratory tests done on a 3 year old boy showed excessive nucleic acid
bases in the urine and negligible activity of hgprtase. Which statement
explains the results obtained?
A.
Purine bases are very insoluble
B.
Low enzyme activity is due to feedback inhibition
C.
Salvage pathway of synthesizing nucleotides is inactive
D.
Young cells utilize the in vivo method for synthesis
Application
Ref. Harper's Illustrated Biochemistry 27th ed., p. 308, par. 1
434.
In myocardial infarction where a major coronary artery is occluded,
ischemia results and a lowered oxygen supply diminishes the amount of
ATP available in the cell. This effect on cellular energy is due to:
A. Release of enzymes
B. Lack of oxygen which slows down the mitochondrial electron
transport chain
C. Inhibition of substrate level phosphorylation
D. Kind of occlusion
58
Application
Ref. Textbook of Biochemistry, 5th ed., Thomas Devlin, p. 591,
Clinical Correlation
435.
The patient was diagnosed to be suffering from megaloblastic anemia.
Results of urinary tests showed 10 mg of formiminoglutamic acid
(FIGLU), making the physician conclude that the patient has folic acid
deficiency. Which statement explains the diagnosis given?
A. Megaloblastic anemia is a unique feature of folic acid deficiency
B. Transfer of formimino group of histidine to tetrahydrofolate does
not take place and figlu is excreted
C. Figlu is a metabolite of folate
D. One carbon compounds are carried by folic acid
Application
Ref. Harper's Illustrated Biochemistry 27th ed.,
p. 254 par. 6
436.
This is a correct statement regarding myoglobin:
A. A monomeric protein
B. Exhibits cooperativity
C. A tetrameric protein
D. Transports oxygen
MPL= 0.5(page:40)
437.
This is a correct statement regarding hemoglobin:
A. Stores oxygen
B. Exhibits cooperativity
C. A stores CO2
D. A non-heme protein
MPL = 0.5 page: 40
438.
The globin chains in hemoglobin and myoglobin functions as/to:
A. oxygen transporter
B. Provides hindered environment for the heme iron
C. inhibits allosteric modulators
D. suit the proteins’ physiologic roles
MPL= 0.5 page: 41
439.
Allosteric properties of hemoglobin is a result if its:
A. Primary structure
B. Quarternary structure
C. Tertiary structure
D. Heme moiety
MPL = 0.5 page 42
440.
The color of heme is due to its:
A. Conjugated ring structure
B. Substituent side chains
C. Cyclic Structure
D. Iron group
MPL = 1.0 p. 271
441.
The key regulatory enzyme in heme biosynthesis is:
A. ALA dehydratase
B. ALA synthase
C. Heme oxygenase
D. UDP-glucuronyltransferase
59
MPL =0.5
p. 272
442.
Sugar with glycemic index less than 1:
A. Glucose
B. Fructose
C. Galactose
D. Lactose
MPL = 1.0 p. 474
443.
Foods with low glycemic index:
A. cause less fluctuation in insulin secretion
B. Not ideal in treating diabetes mellitus
C. examples are starch
D. Glucose have glycemic index more than 1
MPL = 0.5 p. 474
444.
Enzyme that catalyzes hydrolysis of starch:
A. Lipase
B. Amylase
C. protease
D. HCL
MPL = 1.0 p. 474
445.
Clinical manifestation of lactose intolerance is due to:
A. Lactose presence in the large intestines
B. Bacterial fermentation of glucose
C. Bacterial fermentation of galactose
D. Lactase oversecretion of the small intestines
MPL = 0.25 p. 475
446.
A sugar that is absorbed in the small intestine by active transport:
A. Maltose
B. Glucose
C. Fructose
D. Lactose
MPL = 1.0
447.
This is the process of forming glucose from noncarbohydrate precursors
like lactate, amino acids and glycerol:
A. glycolysis
B. gluconeogenesis
C. glycogenesis
D. cori cycle
MPL= 0.5
p. 133
448.
Muscles glycogen cannot contribute directly to plasma glucose because it
lacks this enzyme:
A. Glucose-6-phosphate dehydrogenase
B. Phosphoglucomutase
C. Glucose-6-phosphatase
D. Glucokinase
MPL=0.75 p. 142
60
449.
Nearly all products of digestion of carbohydrates, fats and proteins are
metabolized to what common metabolite?
A. Acetyl CoA
B. pyruvate
C. Glucose-6-phosphate
D. lactate
MPL=0.5
144
450.
It is referred to as the final common pathway for the oxidation of
carbohydrates, lipids and proteins:
A. Embden-Meyerhof pathway
B. Hexose monophosphate shunt
C. Glucuronic acid pathway
D. Citric acid cycle
MPL=1.0
p. 145
451.
These two amino acids yield only Acetyl CoA on oxidation, and hence
cannot be used for gluconeogenesis:
A. Glycine and Lysine
B. Lysine and leucine
C. Valine and Leucine
D. Proline and Glycine
MPL=0.5
p. 139
452.
Interconversion of Glycine and Serine requires what coenzyme?
A. Niacin
B. Riboflavin
C. Pyridoxine
D. Folic acid
MPL=1.0
p. 257
453.
A major role of this vitamin is in the control of cell differentiation and
turnover:
A. Vit A
B. Vit C
C. Vit E
D. Vit D
MPL=0.75 p. 491
454.
This is the earliest sign of Vit A deficiency:
A. Inability to adapt to dim light
B. loss of sensitivity to green light
C. keratomalacia
D. keratinization of the lining of epithelial cells
MPL=0.5
p. 491
455.
What is the most important function of lipid in the adipose tissue and
around internal organs?
A. as electrical insulator
B. as thermal insulator
C. as source of energy
D. for lipid transport
MPL:75%
p. 121
61
456.
Which among the following is a complex lipid composed of fatty acid and
shingosine and carbohydrate?
A. fats
B. oil
C. phospholipids
D. glycolipids
MPL:50%
p. 121
457.
Which eicosanoids promotes bronchoconstriction and play a part in
asthma?
A. prostaglandins
B. leukotienes
C. Thromboxanes
D. Prostacyclin
MPL:50%
p. 122
458.
What is the process involved in the manufacture of margarine?
A. Hydrogenation
B. Hydroxylation
C. Dehydrogenation
D. Oxidation
MPL:50%
p. 123
459.
Which is considered as transcriptionally active chromatin
A. euchromatin
B. heterochromatin
C. constitutive chromatin
D. telomerase
MPL:75%
p. 324
460.
This is the noncoding intervening sequence
A. exon
B. intron
C. kinetochore
D. telomerase
MPL:75%
p. 324
461.
This is the enzyme responsible DNA unwinding during replication
A. DNA ligase
B. Helicase
C. DNA polymerase
D. Topoisomerase
MPL:75%
p. 336
462.
Eukaryotic RNA polymerase II synthesizes:
A. rRNA
B. mRNA
C. tRNA
D. snRNA
MPL:25%
p. 351
463.
What is the key tool in recombinant DNA research?
A. taq polymerase
B. restriction enzymes
C. Deoxyribonuclease
D. Primase
MPL:50%
p. 403
62
464.
The total DNA of a cell or tissue.
A. Genomic DNA library
B. cDNA library
C. introns
D. exons
MPL:50%
p. 408
465.
Immunoglobulin are mostly synthesized in the:
A. T-helper cell
B. Natural killer cells
C. Plasma cells
D. macrophages
MPL:1.0
p. 599
466.
Not true of an immunoglobulin:
A. it is made up of 2 L chains and 2 H chains
B. it has a Y or T shape
C. it has 4 variable regions
D. it has 2 constant regions in its entire structure
MPL:0.25
pp. 599-600
467.
True of the variable region of an immunoglobulins:
A. it is located at the middle most part of the light and heavy chains
B. it is where the CDRs (complementary determining regions)
are located
C. may form two or three antibodies which are identical to each
other.
D. there are no hypervariable regions.
MPL:0.25
p. 601
468.
The part of the antibody that binds to the antigen:
A. Fa,b
B. The antigen determinant site
C. The N-terminal of both the light and the heavy chain
D. The hypervariable regions of both the L and H chains
MPL:0.5
p. 601
469.
The part of the antigen that binds to the antibody:
A. Fa,b
B. The antigen determinant site
C. The N-terminal of both the light and the heavy chain
D. The hypervariable regions of both the L and H chains
MPL:0.5
p. 601
470.
A transcription factor involved in the stimulation of acute phase proteins
synthesis, is nuclear factor __________,
A. Gamma-B
B. Kappa-B
C. Alpha-B
D. Omega-B
MPL:0.25
p. 591
63
471.
The C-reactive protein is so called as such, because it binds with the
_________,
of Pnuemococci, haptoglobin, α, acid glycoproteises:
A. C-monomers
B. C-dimers
C. C-polypeptides
D. C-polysaccharides
MPL:0.25
p. 590
472.
The proteins that stores iron in the cells, is:
A. hemosiderin
B. hemochromatine
C. ferritin
D. . transferring
MPL:0.25
p. 594
473.
The most potent activator of platelet aggregation in thrombus formation is:
A. Collagen
B. TXA
C. ADP
D. fibrinogen
MPL:0.25
p.; 609
474.
A type of thrombus that is formed in arteries is:
A. disseminated fibrin deposits
B. white thrombus
C. red thrombus
D. yellow thrombus
MPL:1.0
p. 606
475.
The chief constituent of fibrous part of plants, is:
A. Starch
B. Cellulose
C. Chitin
D. Peptidoglycan
MPL:1.0
p. 116
Which of the following carbohydrates do not have a structural functions?
A. Cellulose
B. Mucin
C. Chitin
D. Peptidoglycan
MPL:0.25
p. 119
476.
The de novo pathway of purine biosynthesis utilized______ATPs:
A. 7
B. 8
C. 6
D. 5
MPL:0.5
p. 305
477.
The ultimate hydrogen donor in deoxyribonucleotides formation is:
A. thioredoxin
B. NADPH
C. dihydrofolate
D. NADH
MPL:0.5
p. 306
64
478.
Which of the following coenzymes is involved in electron transfer?
A. NAD
B. biotin
C. pyridoxal phosphate
D. tetrahydrofolic acid
MPL:0.5
p. 49
479.
Which of the following statements about noncompetitive inhibitor is true?
A. it is a structural analog of the normal substrate
B. it binds to the active site of enzyme
C. Km is not change but Vmax is decreased
D. It can be reversed by adding more substrate
MPL:0.25
p. 68
480.
Enzymes that are activated through covalent modification usually involved
75
phosphorylation of this amino acid residue in the active site.
A. phenylalanine
B. tryptophan
C. lysine
D. serine
MPL:0.5
p. 75
481.
Carbon monoxide from vehicle exhaust inhibit mitochondrial ATP
production by,
A. decreasing overall potential of the electron transport chain
B. uncoupling oxidative phosphorylation
C. disturbing proton gradient across intermembranous space and
matrix
D. binding to cytochrome oxidase
482.
Oxidation is the,
A. loss of electrons
B. gain of electron
C. gain of protons
D. digestion of energy foods
483.
The final electron acceptor is,
A. carbon dioxide
B. oxygen
C. water
D. hydrogen
484.
Which of the following is responsible for the polarity of a strand of nucleic
acid?
A. acid
B. base
C. sugar
D. phosphate
485.
Nucleic acids are polymeric macromolecules composed of units of,
A. nitrogenous bases
B. nucleotides
C. negatively charge phosphates
D. nucleosomes
65
486.
What is the bond that links nucleotides together to form a strand of nucleic
acid?
A. glycosidic bond
B. phosphodiester bond
C. phosphoester bond
D. hydrogen bond
487.
Which vitamin in the diet is required for de novo pathway of adenosine
monophosphate synthesis and whose deficiency may lead to purine
deficiency, a rare disease in humans?
A. folic acid
B. nicotinic acid
C. pantothenic acid
D. vitamin C
488.
Which of the following organs/tissues is capable of de novo purine
biosynthesis?
A. brain
B. erythrocytes
C. polymorphonuclear leukocytes
D. liver
489.
The lipooxygenase pathway synthesizes:
A. Prostaglandins
B. Leukotrienes
C. Thromboxanes
D. Lipid peroxides
490.
Ketone bodies are oxidized for ATP generation via:
A. Beta-oxidation pathway
B. Cyclooxygenase pathway
C. Lipoxygenase pathway
D. Kreb’s citric acid cycle
491.
Glycerol phosphate, the backbone of TAG, is synthesized in adipose
tissue by:
A. phosphorylation of glycerol by glycerol kinase
B. phosphorylation of diacylglycerol
C. reduction of dihydroxyacetone phosphate
D. glycerol dehydrogenase
492.
Which of the following is not true of the beta-oxidation of palmitic acid?
A. occurs in the matrix of mitochondria
B. two oxidation steps generate 5 mols of ATP per cycle
C. cleavage reaction occurs between the beta and gamma
carbons
D. the cell conserves 129 moles ATP per mole of palmitic acid
493.
Glycolysis, glycogenesis and lipogenesis are stimulated by this hormone:
A. glucagons
B. cortisol
C. insulin
D. epinephrine
66
494.
This hormone is synthesized and released immediately in its final form:
A. aldosterone
B. insulin
C. T3
D. DHT
MPL:1.0
495.
Hormone derived from cholesterol
A. TSH
B. 1,25(OH)2-D3
C. Tyroxine
D. GH
MPL:1.0
p. 447
496.
Which sugar derivatives polymerize to form structural polysaccharide?:
A. Sugar phosphate and sugar acid
B. Sugaramino acid and sugar sulfate
C. Sugar acid and sugar sulfate
D. Sugar amino and sugar acid
497.
Digestability of sugar depends on the presence of this glycosidic linkage:
A. Alpha
B. Beta
C. Gamma
D. Theta
498.
Which of the following disaccharides is best suited to provide the
structural sugars?:
A. Sucrose
B. Lactose
C. Mannose
D. Cellubiose
499.
Which of the following is a saturated fatty acid?:
A. Oleic
B. Linoleic
C. Arachidonic
D. Palmitic
500.
Which of the following is a phospholipid?:
A. Ceramide
B. Cholesterol
C. Cephalin
D. Corticosteroid
Which of the following is a fatty acid?:
A. Succinic acid
B. Prostanoic acid
C. Glutamic acid
D. Butyric acid
501.
Which of the following backbone will make the lipid saponifiable ?:
A. Isoprene nucleus
B. Steroid nucleus
C. Sphingosine
D. Prostanoic acid
67
502.
The presence of which fatty acid will make the membrane more fluid:
A. Myristic
B. Palmitic
C. Stearic
D. Lignoceric
503.
Which of the following lipids will stabilize membrane?:
A. Cardiolipin
B. Cortisol
C. Cholesterol
D. Ceramide
504.
Which of these is an essential amino acid?:
A. Serine
B. Tyrosine
C. Isoleucine
D. Glutamine
505.
The simplest amino acid is:
A. Cysteine
B. Glycine
C. Serine
D. Valine
506.
An example of an aromatic amino acid is:
A. Serine
B. Aspartic
C. Glutamic
D. Tryptophan
507.
A protein exposed to denaturing agent becomes:
A. Antigenic
B. Inert
C. Undigestible
D. Reactive
508.
The linkage between the nucleotides:
A. Glycosidic
B. Peptide
C. Methene
D. Phosphodiester
509.
An important bond in base pairing:
A. Covalent
B. Glycosidic
C. Hydrogen
D. Phosphodiester
510.
DNA strands that show high melting temperature contains:
A. High carbohydrate residues
B. High histone content
C. High G-C pairs
D. High supercoiling
68
511.
Which purine analogue is useful in cancer chemotherapy?:
A. Propyl thiuracil
B. 1,3,7 trimethylxanthine
C. 6 mercaptopurine
D. 4 hydroxypyrazolopyramidine
512.
Which vitamin is a purine derivative?:
A. Cyanocobalamine
B. Inositol
C. Carotene
D. Niacin
513.
Enzyme that hydrolyzes glycogen:
A. Hexokinase
B. Phosphorylase
C. Aconitase
D. Transketolase
514.
A metabolic intermediate leading to glycogenesis, pentose phosphate
pathway and glycolysis:
A. Acetyl CoA
B. Citric acid
C. Glucose 6 phosphate
D. Pyruvate
515.
This metabolite when transaminated leads to sugar amine pathway:
A. Fructose 6 phosphate
B. Glyceraldehyde 3 phosphate
C. Glucose 1 phosphate
D. Ribose 5 phosphate
516.
This metabolite leads to the production of NAD in Embden Meyerhoff
pathway:
A. Acetyl CoA
B. Glyceraldehyde 3 phosphate
C. Lactic acid
D. Pyruvate
517.
This pathway sustains the energy requirement of mature RBC.
A. Kreb’s cycle
B. Pentose phosphate
C. Glycolysis
D. Anaplerotic
518.
This pathway provides NADPH for lipogenesis
A. Kreb’s cycle
B. Phosphogluconic
C. Uronic
D. Cori
519.
In the intestinal mucosal cells, triglyceride is:
A. Hydrolyzed
B. Esterified
C. Synthesized
D. Absorbed
69
520.
Lipolysis will be formed with increased levels of this hormone:
A. Insulin
B. Epinephrine
C. Cortisol
D. Esatrogen
521.
In beta oxidation, for every break of an ester bond (saturated) this much
amount of ATP is generated:
A. 23
B. 3
C. 5
D. 12
522.
In fasting and starvation the level of these metabolites are increased:
A. Oxysteroid
B. Ketone bodies
C. Thyroxine
D. Insulin
523.
Arachidonic acid deficiency usually do not occur because of this enzyme:
A. Polymerase
B. Kinase
C. Desaturase
D. Ligase
524.
Amino acids are transported intracellularly using this cycle:
A. Kreb’s cycle
B. Kreb’s – hanseleit cycle
C. Cori cycle
D. Gamma glutamyl cycle
525.
The major product of nitrogen catabolism in humans is:
A. Uric acid
B. Creatinine
C. Urea
D. Ammonia
526.
What urinary metabolite is useful in the assessment catecholamine
induced hypertension?:
A. Hydroxyindole acetic acid
B. Phenylpyruvic acid
C. Vanillyl mandylic acid
D. Paraamino hippuric acid
527.
The starting materials for porphyrin biosynthesis are:
A. Acetyl CoA and serine
B. Malonyl CoA and glutamine
C. Succinyl CoA and glycine
D. Amino acyl and methionine
528.
Which of these is the key regulatory enzyme in heme biosynthesis?:
A. Ferrochelatase
B. ALA synthase
C. Pyrollase
D. Uroporphyrin decarboxylase
70
529.
What is this organic acid that is used to conjugate bilirubin?:
A. Propionic acid
B. Glucuronic
C. Acetic acid
D. Butyric acid
530.
Which of these enzyme deficiency produces abdominal pain with out skin
manifestation?:
A. Coproporphyrinogen oxidase
B. Uroporphyrin synthase
C. Ferrochelatase
D. Pyrollase
531.
Activity exhibited by enzymes when fully saturated:
A. Km
B. Vmax
C. ½ Vmax
D. Vo
532.
CPK elevation is suggestive of:
A. Hepatitis
B. Cardiomyopathy
C. Pancreatitis
D. Bone tumor
533.
Pyruvate kinase deficiency produces:
A. Hypoglycemia
B. Hemolytic anemia
C. Hypercholesterolemia
D. Mal absorption
534.
Which of these is a pyrimidine excretory metabolite?:
A. Ammonia
B. Urea
C. Uric acid
D. Allantoin
535.
Which of these enzymes is important in DNA sequencing?
A.
B.
C.
D.
536.
Ligase
Polymerase
Exonuclease
Kinase
A very anxious student hyperventilates an hour before the biochemistry
board exam. What is the effect of hyperventilation on his Pco 2 and blood
pH?
A. Pco2 decreases and pH increases.
B. Pco2 decreases and pH decreases.
C. Pco2 increases and pH decreases.
D. Pco2 increases and pH increases.
Ans. A
Ref: Harper’s Biochemistry 24th ed. Pp. 18-19
MPL: 0.25
71
537.
A protein rich in which of the following amino acids will provide the
greatest buffering capacity at physiologic pH?
A. Valine
B. Aspartic acid
C. Lysine
D. Histidine
Ans. D
Ref: Harper’s Biochemistry 24th ed. p.26
MPL: 0.75
538.
Which of the following laboratory results would best indicate metabolic
alkalosis?
A. Increased Pco2, increased pH, increased HCO3.
B. Normal Pco2, increased pH, decreased HCO3.
C. Increased Pco2, increased pH, normal HCO3.
D. Decreased Pco2, increased pH, normal HCO3.
Ans. A
Ref: Harper’s Biochemistry 24th ed. p.304
MPL: 0.75
539.
What is the property of water that will contribute the most to its ability to
dissolve compounds?
A. Very few interactive forces in its structure.
B. Hydrogen bond formation between water and long-chain fatty acids.
C. Covalent bond formation between salt and water.
D. Hydrogen bond formation between water and biochemical
molecules.
Ans. D
Ref: Harper’s Biochemistry 24th ed. p.15-17
MPL: 0.75
540.
A 3-year old child is diagnosed to have Hurler’s syndrome. The diagnosis
of Hurler’s syndrome is MOST efficiently made by analyzing the patient’s
DNA for:
A. A region of DNA that does not encode RNA.
B. Alternative forms of the L-Iduronidase gene.
C. The entire set of genes in one leukocyte.
D. A nucleotide substitution in the L-Iduronidase gene.
Ans. B
Ref: Harper’s Biochemistry 24th ed. p. 678
MPL: 0.25
541.
Which of the following statements regarding a double-helical molecule of
DNA is TRUE?
A. All hydroxyl groups of pentoses are involved in linkages.
B. Bases are perpendicular to the axis.
C. Each strand is identical.
D. Each strand replicates itself.
Ans. B
Ref: Harper’s Biochemistry 24th ed. p.386
MPL: 0.75
72
542.
A newborn baby has a sibling with sickle cell anemia and is at risk for this
disease. The appropriate diagnostic test for sickle cell anemia in this baby
would include which of the following?
A. DNA amplification
B. Hemoglobin antibodies
C. Red cell counting
D. DNA fingerprinting
Ans. A
Ref: Harper’s Biochemistry 24th ed. p. 468
MPL: 0.25
543.
Which of the following would BEST describe polymorphism?
A. One phenotype, multiple genotypes
B. Non-random allele association
C. One locus, multiple abnormal alleles
D. One locus, multiple normal alleles
Ans. D
Ref: Harper’s Biochemistry 24th ed. p. 476
MPL: 0.25
544.
What is that process that occurs at the 5-position of Cytidine and is often
correlated with gene inactivation?
A. Gene conversion
B. Sister chromatid exchange
C. Gene rearrangement
D. DNA methylation
Ans. D
Ref: Harper’s Biochemistry 24th ed. p.466
MPL: 0.75
545.
Restriction Fragment Length Polymorphism (RFLP) analysis can only be
used to follow the inheritance of a genetic disease if:
A. The disease-causing mutation is at or closely linked to an
altered restriction site.
B. Proteins of mutated and normal genes migrate differently upon gel
electrophoresis.
C. Mutations are outside restriction sites so that cleaving still occurs.
D. mRNA probes are used in combination with antibodies.
Ans. A
Ref: Harper’s Biochemistry 24th ed. p. 478
MPL: 0.25
546.
The chromosomes of mammalian cells are 20 times as large as those of
Escherichia coli. How can replication of mammalian chromosomes be
carried out in just a few minutes?
A. The higher temperature of mammalian cells allows for an
exponentially higher replication rate.
B. Hundreds of replication forks work simultaneously on each
piece of chromosomal DNA.
C. Many RNA polymerases carry out replication simultaneously on
chromosomal DNA.
D. The presence of histones speeds up the rate of chromosomal DNA
replication.
73
Ans. B
Ref: Harper’s Biochemistry 24th ed. p. 407
MPL: 0.75
547.
Following ultraviolet damage to DNA in the skin, which of the following is
MOST likely to occur?
A. A specific nuclease detects damaged areas.
B. Purine dimmers are formed.
C. Both strands are cleaved.
D. Endonuclease removes the damaged strand.
Ref: Harper’s Biochemistry 24th ed. p. 814
MPL: 0.75
548.
Which of the following statements about the genetic code is the MOST
accurate?
A. Information is stored as sets of dinucleotide repeats called codons.
B. The code is degenerate.
C. There are 64 codons, all of which code for amino acids.
D. Information is stored as sets of trinucleotide repeats called codons.
Ref: Harper’s Biochemistry 24th ed. p. 433-434
MPL: 0.75
549.
Which of the following is provided by the Northern Blot Analysis?
A. Detection of specific base pairs.
B. Detection of DNA molecules.
C. Detection of RNA molecules.
D. Detection of proteins.
Ref: Harper’s Biochemistry 24th ed. p. 472-473
MPL: 1.0
550.
In contrast to DNA polymerase, RNA polymerase:
A. Fills in the gap between the Okazaki fragments.
B. Works only in a 5’ to 3’ direction.
C. Edits as it synthesizes.
D. Synthesizes RNA primer to initiate DNA synthesis.
Ref: Harper’s Biochemistry 24th ed. p. 418
MPL: 0.75
551.
What is the function of a promoter site on DNA?
A. Transcribes repressor
B. Initiates transcription
C. Codes for RNA polymerase
D. Regulates termination
Ref: Harper’s Biochemistry 24th ed. p. 420
MPL: 1.0
74
552.
553.
The consensus sequence 5’-TATAAAA-3’ found ineukaryotic genes is
almost similar to a consensus sequence observed in prokaryotes. The
consensus sequence is important as the:
A. Only site of binding of RNA polymerase II.
B. Promoter site for all RNA polymerases.
C. Termination site for RNA polymerase II.
D. First site of binding of a transcription factor for RNA
polymerase II.
Ref: Harper’s Biochemistry 24th ed. pp. 421-422
MPL: 0.75
The so-called “caps” of RNA molecules:
A. Allow tRNA to be processed.
B. Occur at the 3’-end of tRNA.
C. Are composed of poly A.
D. Are unique to eukaryotic mRNA.
Ref: Harper’s Biochemistry 24th ed. p. 428
MPL: 1.0
554.
In bacterial RNA synthesis, what is the function of the “rho” factor?
A. It binds catabolite repressor to the promoter region.
B. It increases the rate of RNA synthesis.
C. It eliminates the binding of RNA polymerase to the promoter.
D. It participates in the proper termination of transcription.
Ref: Harper’s Biochemistry 24th ed. p. 419
MPL: 1.0
555.
Which of the following statements correctly describes the nucleolus of a
mammalian cell?
A. It differs from that found in bacterial cells in that histones are
present.
B. It may contain hundreds of copies of genes for different types
of ribosomal RNAs.
C. It synthesizes 5S rRNA.
D. It synthesizes all ribosomal RNA primary transcripts.
Ref; Harper’s Biochemistry 24th ed. p. 428
MPL: 0.75
556.
Which of the following statements correctly describes the synthesis of
mammalian mRNA?
A. Each mRNA often encodes several different proteins.
B. Several different genes may produce identical mRNA
molecules.
C. Mammalian mRNA undergoes minimal modification during its
maturation.
D. The RNA sequence transcribed from a gene is identical to the
mRNA that exits from nucleus to cytoplasm.
Ref: Harper’s Biochemistry 24th ed. p. 427
MPL: 0.75
75
557.
Certain amino acids are not part of the primary structure of proteins but
are modified after translation. In scurvy, which amino acid that is normally
part of collagen is not synthesized?
A. Hydroxytryptophan
B. Hydroxytyrosine
C. Hydroxyhistidine
D. Hydroxyproline
Ref: Harper’s Biochemistry 24th ed. pp. 668-669
MPL: 1,0
558.
Immunoglobulin G (IgG) molecules can be characterized by which of the
following statements?
A. They are maintained at a constant level in the serum.
B. They contain nucleic acids.
C. They contain mostly carbohydrates.
D. They can be separated into subunits with a reducing agent and
urea.
Ref: Harper’s Biochemistry 24th ed. p.716
MPL: 1.0
559.
Which of the following proteolytic enzymes is activated by acid hydrolysis
of the proenzyme form?
A. Trypsin
B. Elastase
C. Pepsin
D. Carboxypeptidase
Ref: Harper’s Biochemistry 24th ed. p.636
MPL: 1.0
560.
Which of the following amino acids is ionizable in proteins?
A. Leucine
B. Histidine
C. Valine
D. Alanine
Ref: Harper’s Biochemistry 24th ed. p. 294
MPL: 0.75
561.
The oxygen carrier of muscle is the globular protein myoglobin. Which of
the following amino acids is highly likely to be localized within the interior
of the molecule?
A. Arginine
B. Aspartic acid
C. Glutamic acid
D. Valine
Ref: Harper’s Biochemistry 24th ed. p.53
MPL: 0.75
76
562.
Which of the following statements concerning immunoglobulins is most
accurate?
A. IgE is the principal antibody in the serum.
B. The heavy chains are similar in each class of immunoglobulin.
C. The constant regions of the heavy chains are the same in each
class of immunoglobulin.
D. IgE is the major immunoglobulin found in external secretions.
Ref: Harper’s Biochemistry 24th ed. p.718
MPL: 1.0
563.
Under normal conditions in the blood, which of the following amino acid
residues of albumin is neutral?
A. Arginine
B. Aspartate
C. Glutamine
D. Histidine
Ref: Harper’s Biochemistry 24th ed. p. 294
MPL: 0.75
564.
Which of the following carbohydrates would be most abundant in a diet of
strict vegetarians?
A. Amylose
B. Lactose
C. Cellulose
D. Glycogen
Ref: Harper’s Biochemistry 24th ed. p. 141
MPL: 1.0
565.
The major metabolic product under normal circumstances by erythrocytes
and by muscle cells during intense exercise is recycled through the liver in
the Cori cycle. The metabolite is:
A. Oxaloacetate
B. Glycerol
C. Alanine
D. Lactate
Ref: Harper’s Biochemistry 24th ed. p. 200-201
MPL: 0.75
566.
Following a fad diet of skimmed milk and yogurt, an adult female patient
experiences abdominal distention, nausea, cramping, and pain followed
by watery diarrhea. This set of symptoms is observed each time the said
meal is consumed. A most likely diagnosis is:
A. Steatorrhea
B. Lactase deficiency
C. Sialidase deficiency
D. Lipoprotein deficiency
Ref: Harper’s Biochemistry 24th ed. p. 645
MPL: 0.1
77
567.
Which of the following enzymes catalyzes the high-energy
phosphorylation of substrates during glycolysis?
A. Pyruvate kinase
B. Phosphoglycerate kinase
C. Triosephosphate isomerase
D. Glyceraldehyde-3-phosphate dehydrogenase
Ref: Harper’s Biochemistry 24th ed. p.176
MPL: 1.0
568.
Which of the following enzymes is common to both glycolysis and
gluconeogenesis?
A. Pyruvate carboxylase
B. Hexokinase
C. Phosphoglycerate kinase
D. Fructose 1,6-bisphosphatase
Ref: Harper’s Biochemistry 24th ed. p.194
MPL: 0.75
569.
During the first week of a diet of 1500 calories per day, the oxidation of
glucose via glycolysis in the liver of a normal 59-kg woman is inhibited by
the lowering of which of the following?
A. Citrate
B. ATP
C. Ketone bodies
D. Fructose-2,6-bisphosphatase
Ref: Harper’s Biochemistry 24th ed. p.176
MPL: 0.75
570.
Familial fructokinase deficiency causes no symptoms because:
A. Hexokinase can phosphorylate fructose.
B. Most tissues utilize fructose.
C. Liver fructose-1-phosphate aldolase is still active.
D. Excess fructose spills into the bowel and is eliminated in the feces.
Ref: Harper’s Biochemistry 24th ed. p. 214
MPL: 0.75
571.
Which of the following enzymes catalyzes phosphorylation with the use of
inorganic phosphate?
A. Hexokinase
B. Phosphofructokinase
C. Phosphoglycerate kinase
D. Glyceraldehyde-3-phosphate dehydrogenase
Ref: Harper’s Biochemistry 24th ed. p. 179
MPL: 0.75
572.
After a well-rounded breakfast, which of the following would be expected
to occur?
A. Increased activity of pyruvate carboxylase
B. Decreased activity of acetyl CoA carboxylase
C. Decreased rate of glycogenesis
D. Decreased rate of protein synthesis
78
Ref.
MPL:
573.
Harper’s Biochemistry 24th ed. p. 179
0.75
Contraction of skeletal muscle is initiated by the binding of calcium to:
A. Tropomyosin
B. Troponin
C. Myosin
D. Actomyosin
Ref. Harper’s Biochemistry 24th ed. p. 687-688
MPL: 0.75
574.
Which of the following statements correctly describes transport of oxygen
by hemoglobin?
A. Oxygen binds to hemoglobin more avidly than does carbon
monoxide.
B. The binding of oxygen to hemoglobin causes a valence change in
the iron of the heme moiety.
C. Each of the four heme moieties binds oxygen independently
D. The plot of percentage of oxygen bound versus oxygen
pressure is sigmoid in shape
Ref. Harper’s Biochemistry 24th ed. p. 436
MPL: 0.75
575.
Which of the following mutations would produce a severe form of
thalassemia?
A. Deletion of one α-globin locus
B. Deletion of one β-globin locus
C. Oxidation of heme groups to produce methemoglobin
D. Altered RNA processing at both
Ref. Harper’s Biochemistry 24th ed. p. 734
MPL 0.75
576.
An increased affinity of hemoglobin for oxygen may result from which of
the following?
A. Initial binding of oxygen to one of the four sites available in
each deoxyhemoglobin molecule
B. High pH
C. High carbon dioxide levels
D. High 2,3-bisphosphoglycerate levels within erthrocytes
Ref. Harper’s Biochemistry 24th ed. p. 436
MPL 0.75
577.
The functions of many enzymes, membrane transporters, and other
proteins can be quickly activated or deactivated by phosphorylation of
specific amino acid residues catalyzed by enzymes called:
A. Cyclases
B. Kinases
C. Phosphatases
D. Proteases
Ref. Harper’s Biochemistry 24th ed. p. 64-65
MPL 0.75
79
578.
The Vmax of the enzyme is the:
A. Reciprocal of the absolute value of the intercept of the curve with
the x-axis
B. Reciprocal of the absolute value of the intercept of the curve
with the y-axis
C. Absolute value of the intercept of the curve with the x-axis
D. Slope of the curve
Ref. Harper’s Biochemistry 24th ed. p. 75-90
MPL 0.75
579.
In the study of enzymes, the sigmoidal plot of substrate concentration
versus reaction velocity may indicate:
A. Michaelis-Menten Kinetics
B. Myoglobin binding oxygen
C. Cooperative binding
D. Competitive inhibition
Ref. Harper’s Biochemistry 24th ed. p. 75, 90
MPL 1.0
580.
A non competitive inhibitor of an enzyme:
A. Increases Km with no or little change in Vmax
B. Decreases Km and decreases Vmax
C. Decreases Vmax
D. Increases Vmax
Ref. Harper’s Biochemistry 24th ed. p. 86-88
MPL 0.75
581.
Which of the following statements correctly describes allosteric enzymes?
A. Effectors may enhance or inhibit substrate binding
B. They are not usually controlled by feedback inhibition
C. The regulatory site may be catalytic site
D. Michaelis-Menten kinetics describe their activity
Ref. Harper’s Biochemistry 24th ed. p. 104-105
MPL 0.75
582.
Which of the following enzymes is regulated primarily through allosteric
interaction?
A. Chymotrypsin
B. Pyruvate dehydrogenase
C. Glycogen phosphorylase
D. Aspartate transcarbamoylase
Ref. Harper’s Biochemistry 24th ed. p. 104-105
MPL 0.75
583.
Which of the following statements about flavoproteins is TRUE?
A. They are not oxidized by coenzymeQ
B. They receive electrons from cytochrome P450 in liver mitochondria
C. They do not participate in oxidation of NADH dehydrogenises
D. They can be associated with sulfur and nonheme iron
Ref. Harper’s Biochemistry 24th ed. p. 117-118
MPL 0.75
80
584.
Which of the following compounds is a member of the electron transport
chain?
A. Octanoyl carnitine
B. Cytochrome c
C. NADH
D. Palmitoyl carnitine
Ref. Harper’s Biochemistry 24th ed. p. 128
MPL 0.75
585.
All known effects of cyclic AMP in eukaryotic cells result from:
A. Activation of the catalytic unit of adenylate cyclase
B. Activation of synthetases
C. Activation of protein kinase
D. Stimulation of calcium release from the endoplasmic reticulum
Ref. Harper’s Biochemistry 24th ed. p. 165
MPL 0.75
586.
The connection between oxidative phosphorylation and electron transport
is BEST described by:
A. Existence of higher pH in the cisternae of the endoplasmic
reticulum than in the cytosol.
B. Synthesis of ATP as protons into the mitochondrial matrix
along a proton gradient that exist across the inner
mitochondrial membrane
C. Dissociation of electron transport and oxidative phosphorylation
D. Absence of ATPase in the inner mitochondrial membrane
Ref. Harper’s Biochemistry 24th ed. p. 127
MPL 0.75
587.
If all potential sources of ATP production are taken into account, the net
number of ATP molecules formed per molecule of glucose in aerobic
glycolysis is:
A. 2
B. 6
C. 18
D. 36
Ref. Harper’s Biochemistry 24th ed. p. 176-184
MPL 0.75
588.
Which of the following reactions generates ATP/
A. Glucose 6-phosphate to Fructose 6-phosphate
B. Glucose to Glucose 6-Phosphate
C. Fructose 6-phosphate to Fructose 1,6-bisphosphate
D. Phosphoenolpyruvate
Ref. Harper’s Biochemistry 24th ed. p. 176-184
MPL 0.75
81
589.
Which of the following products of triacylglycerol breakdown and
subsequent β-oxidation may undergo gluconeogenesis?
A. Propionyl CoA
B. Acetyl CoA
C. All ketone bodies
D. Some amino acids
Ref. Harper’s Biochemistry 24th ed. p. 284
MPL 0.75
590.
Inhibition of ATP synthesis during oxidative phosphorylation by oligomycin
is thought to be due to:
A. Blocking of the proton gradient between NADH-Q reductase & QH2
B. Blocking of the proton gradient between cytochrome C1 and
cytochrome C
C. Dissociation of cytochrome c from mitochondrial membranes
D. Inhibition of mitochondrial ATPase
Ref. Harper’s Biochemistry 24th ed. p. 701
MPL 0.75
591.
The reactions of the urea cycle occur:
A. In the cytosol
B. In the mitochondrial matrix
C. In the mitochondrial matrix & the cytosol
D. Only in lysosomes
Ref. Harper’s Biochemistry 24th ed. p. 301-380
MPL 0.75
592.
The thyroid hormone thyroxine (T4) is derived from:
A. Threonine
B. Tyrosine
C. Thiamine
D. Tryptophan
Ref. Harper’s Biochemistry 24th ed. p. 534-536
MPL 0.75
593.
Which of the metabolites below is a precursor of tyrosine?
A. L-dihydroxyphenylalanine (DOPA)
B. Dopamine
C. Norepinephrine
D. Phenylalanine
Ref. Harper’s Biochemistry 24th ed. p. 296-297
MPL 0.75
594.
Which clinical laboratory observation is suggestive of Hartnup’s disease?
A. High fecal levels of tryptophan and indole derivatives
B. Elevated plasma tyrosine and methionine levels
C. Elevation of glutamine in blood and urine
D. Extremely high levels of citrulline in urine
Ref. Harper’s Biochemistry 24th ed. p. 322-325
MPL 0.75
82
595.
The important receive group of glutathione in its role as antioxidant is:
A. Serine
B. Sulfhydryl
C. Tyrosine
D. Acetyl CoA
Ref. Harper’s Biochemistry 24th ed. p. 753
MPL 0.75
596.
A newborn develops jaundice that requires laboratory evaluation. Which
of the following porphyrin derivatives is conjugated, reacts directly, and is
a major component of bile?
A. Stercobilin
B. Biliverdin
C. Bilirubin
D. Bilirubin diglucuronide
Ref. Harper’s Biochemistry 24th ed. p. 356
MPL 0.75
597.
Which of the following poryphrins give stools their characteristic brown
color?
A. Biliverdin
B. Urobilinogen
C. Heme
D. Stercobilin
Ref. Harper’s Biochemistry 24th ed. p. 354
MPL 0.75
598.
Ceramide is a precursor to which of the following compounds?
A. Phosphatidyl serine
B. Sphingomyelin
C. Phosphatidyl glycerol
D. Phosphatidyl choline
Ref. Harper’s Biochemistry 24th ed. p. 250-252
MPL 0.75
599.
Humans MOST easily tolerate a lack of which of the following nutrients?
A. Protein
B. Iodine
C. Carbohydrate
D. Lipid
Ref. Harper’s Biochemistry 24th ed. p. 285-286
MPL 0.75
600.
A deficiency in Vitamin B12 causes:
A. Cheilosis
B. Beriberi
C. Pernicious anemia
D. Scurvy
Ref.
Harper’s Biochemistry 24th ed. p. 612-613
83
MPL 0.75
601.
Which of the following vitaminswould MOST likely become deficient in a
person who develops a completely carnivorous lifestyle?
A. Thiamine
B. Niacin
C. Cobalamin
D. Vitamin C
Ref. Harper’s Biochemistry 24th ed. p. 612-613
MPL 0.75
602.
603.
Biotin is involved in which of the following types of reactions?
A. Hydroxlations
B. Carboxylations
C. Decarboxylations
D. Dehydrations
Ref. Harper’s Biochemistry 24th ed. p. 606-607
MPL 0.75
Which of the following conditions would primarily produce a functional
deficiency of Vitamin K?
A. Coumadin Therapy
B. Broad Spectrum antibiotic
C. Premature birth
D. Lack of red meat in the diet
Ref. Harper’s Biochemistry 24th ed. p. 727
MPL 0.75
604.
A 3 month old boy presents with poor feeding & growth, hypotonia, lactic
acidemia, and mild acidosis. The pyruvate to lactate ratio is high, and
there is decreased conversion of pyruvate to Acetyl CoA. Which of the
following may be considered for therapy?
A. Thiamine
B. FFA
C. Biotin
D. Vitamin C
Ref. Harper’s Biochemistry 24th ed. p. 183
MPL 0.25
605.
A 2-year old child presents with cough and bronchitis, growth failure, &
chronic diarrhea with light-colored foul smelling stools. A deficiency of
which vitamin should be considered?
A. Vit B6
B. Vit C
C. Vit A
D. Vit B1
Ref. Harper’s Biochemistry 24th ed. p. 614-617
MPL 0.25
84
606.
An infant presents with prominent forehead, bowing of the limbs, broad &
tender wrists, swelling at the costochondral junction of the ribs, and
irritability. Which of the following treatments is the most appropriate for
this patient?
A. Removal of eggs from diet
B. Milk & Sunlight exposure
C. Diet of baby food containing liver & ground beef
D. Diet of baby food containing leafy vegetables
Ref. Harper’s Biochemistry 24th ed. p. 613
MPL 0.25
607.
Allopurinol effectively treats gout but has NO effect on the severe
neurological symptoms of Lesch-Nyhan patients because it does not:
A. Decrease de Novo purine synthesis
B. Decrease de Novo Pyrimidine syntheis
C. Inhibit xanthine oxidase
D. Increase PRPP levels
Ref. Harper’s Biochemistry 24th ed. p. 378
MPL 0.75
608.
Which of the following would rule out hyperuricemia in a ptient?
A. Lesch-Nyhan synthesis
B. Gout
C. Carbonage phosphate deficiency
D. Xanthine oxidase hyperactivity
Ref. Harper’s Biochemistry 24th ed. p. 381
MPL 0.75
609.
Which of the following contributes nitrogen atom to both purine &
pyrimidine rings?
A. Aspartate
B. Carbonage PO4
C. CO2
D. None of the above
Ref. Harper’s Biochemistry 24th edition P. 376-377
MPL 1.0
610.
What is the end product of fatty acid synthase activity in human?
A. Palmitic acid
B. Palmetoleic acid
C. Linoleic
D. Arachidonic acid
Ref. Harper’s Biochemistry 24th ed. p. 216-223
MPL 0.75
611.
For every 2 moles of free glycerol released by lipolysis of triglycerides in
adipose tissue?
A. 1 mole of glucose can be synthesized via gluconeogenesis
B. 2 moles of triacrylglyceride is released
C. 2 moles of FFA is released
D. 3 moles of Acyl CoA is produced
85
Ref. Harper’s Biochemistry 24th ed. p. 267
MPL 0.75
612.
A 45 year old man ha s a mild heart attack and is placed on diet and
Mevastatin therapy. Which of the following will result in this therapy?
A. Low blood glucose level
B. Low blood LDLs
C. High blood cholesterol
D. Low oxidation of Fa
Ref. Harper’s Biochemistry 24th ed. p. 281
MPL 0.75
86